When can I stop dual antiplatelet therapy in patients with drug-eluting stents?

Article Type
Changed
Thu, 01/03/2019 - 09:43
Display Headline
When can I stop dual antiplatelet therapy in patients with drug-eluting stents?

Stopping dual antiplatelet therapy (DAPT) (eg, clopidogrel plus aspirin) after 3 months is reasonable in patients with stable ischemic heart disease who have a second-generation drug-eluting stent and a high bleeding risk, with stable ischemic disease defined as at least 1 year free of acute coronary syndromes. However, these patients should continue lifelong aspirin monotherapy. Current guidelines suggest that in stable ischemic disease, the risk-benefit ratio may favor an even shorter duration of DAPT than the 6 months currently recommended.1

STABLE ISCHEMIC HEART DISEASE VS ACUTE CORONARY SYNDROME

Percutaneous coronary intervention for stable ischemic heart disease is indicated primarily in patients with angina that persists despite optimal antianginal therapy.

The prognostic implications of DAPT are different in stable ischemic disease than in acute coronary syndromes. The substrate treated by percutaneous intervention in stable ischemic disease is primarily fibrofatty plaque, as opposed to thrombus in acute coronary syndromes.

Percutaneous intervention significantly improves the prognosis in acute coronary syndromes, whereas its impact on overall survival in stable ischemic heart disease is not well documented. Given these differences, our discussion about DAPT in stable ischemic disease cannot be extrapolated to acute coronary syndromes.

BENEFITS OF DAPT

DAPT is mandatory early after drug-eluting stent placement, when the stent continuously releases medication, inhibiting tissue growth within the lumen of the stent.

Endothelialization of the stent normally occurs during the first 7 to 30 days after placement. During this period, the nonendothelialized stent poses a risk of thrombosis, a life-threatening, catastrophic condition with a mortality rate between 9% and 45%.1

Aspirin 75 to 100 mg has been shown to be effective as secondary prevention of atherosclerotic disease and is recommended lifelong in this clinical setting. Adding a thienopyridine reduces the risk of myocardial infarction, stent thrombosis, and death from a cardiovascular event and decreases the incidence of plaque rupture in nonstented coronary vessels. Hence, prevention of these complications provides the rationale for DAPT in this clinical setting.

THERAPY BEYOND 12 MONTHS

Although guidelines have traditionally recommended 12 months of DAPT, the optimal duration is still debated.

A duration beyond 12 months in patients with a history of myocardial infarction was shown to be reasonable in 2 large trials,2,3 while a 2016 review by Bittl et al4 suggested that therapy beyond 12 months in patients with a newer-generation drug-eluting stent could increase the incidence of major bleeding. A detailed discussion of DAPT longer than 12 months is beyond the scope of this article.

EVIDENCE FOR SHORTER DURATION

The results of 5 major trials support shorter duration of DAPT in stable ischemic disease.

The OPTIMIZE5 and RESET6 trials found that 3 months of DAPT was not inferior to 12 months in terms of ischemic and safety end points.

The ISAR-SAFE,7 EXCELLENT,8 and SECURITY9 trials also reported that 6 months of DAPT was not inferior to 12 months for the primary composite end point of death, stent thrombosis, myocardial infarction, stroke, or major bleeding.

However, these trials may have been underpowered to detect a difference in rates of stent thrombosis with shorter-duration DAPT.

CURRENT GUIDELINES

For patients at high bleeding risk, the current guidelines of the American College of Cardiology and American Heart Association, updated in 2016, suggest that it may be reasonable to discontinue DAPT 3 months after drug-eluting stent placement in patients with stable ischemic heart disease, and at 6 months in patients with acute coronary syndrome (class IIb recommendation, level of evidence C).1 These recommendations are based on results of randomized controlled trials showing no difference in the rate of stent thrombosis and composite ischemic events with a shorter duration than with 12 months of therapy.5–10

The evidence for DAPT in stable ischemic disease is based on clopidogrel, with only limited data on ticagrelor.1 To our knowledge, no study to date has evaluated DAPT in this setting for less than 3 months, and further study is needed to address shorter-duration approaches with current-generation drug-eluting stents Since 2017, all coronary stents implanted in the United States have been second-generation stents.

 

 

TOOLS TO HELP DECISION-MAKING

The decision to stop DAPT in a patient at high risk of bleeding requires a careful assessment of the risks and benefits. Risk factors for bleeding include advanced age, history of major bleeding, anticoagulation, chronic kidney disease (serum creatinine level ≥ 2 mg/dL), platelet count 100 × 109/L or lower, and history of stroke.11

Table 1. Risk factors for ischemia, stent thrombosis, and bleeding
A useful approach is to define the risks of stent thrombosis and bleeding (Table 1).1 The DAPT score determines the risk-benefit ratio for long-term DAPT as follows:

  • Age 75 or older: −2 points
  • Ages 65 to 74: −1
  • Age under 65: 0
  • Diabetes mellitus: 1
  • Myocardial infarction at presentation: 1
  • History of percutaneous coronary intervention or myocardial infarction: 1
  • Stent diameter less than 3 mm: 1
  • Paclitaxel drug-eluting stent: 1
  • Current smoker: 2
  • Percutaneous coronary intervention with saphenous vein graft: 2
  • Congestive heart failure or left ventricular ejection fraction less than 30%: 2.

A score of 2 or greater favors continuing DAPT, as it indicates higher ischemic risk. A score less than 2 favors discontinuing DAPT, as it indicates higher bleeding risk.1,2

IF BLEEDING RISK IS HIGH

Preventing and controlling bleeding associated with DAPT is important. The gastrointestinal tract is the most common site of bleeding.

Aspirin inhibits prostaglandin synthesis, leading to disruption of the protective mucous membrane. Therefore, a proton pump inhibitor should be started along with DAPT in patients at high risk of gastrointestinal bleeding.

If a patient’s bleeding risk significantly outweighs the risk of stent thrombosis, or if active hemorrhage makes a patient hemodynamically unstable, antiplatelet therapy must be stopped.1

FACING SURGERY

For patients with a drug-eluting stent who are on DAPT and are to undergo elective noncardiac surgery, 3 considerations must be kept in mind:

  • The risk of stent thrombosis if DAPT needs to be interrupted
  • The consequences of delaying the surgical procedure
  • The risk and consequences of periprocedural and intraprocedural bleeding if DAPT is continued.

Because clinical evidence for bridging therapy with intravenous antiplatelet or anticoagulant agents is limited, it is difficult to make recommendations about stopping DAPT. However, once bleeding risk is stabilized, DAPT should be restarted as soon as possible.1

CURRENT RESEARCH

Several trials are under way to further evaluate ways to minimize bleeding risk and shorten the duration of DAPT.

A prospective multicenter trial is evaluating 3-month DAPT in patients at high bleeding risk who undergo placement of an everolimus-eluting stent.11 This study is expected to be completed in August 2019.

Another strategy for patients at high bleeding risk is use of a polymer-free drug-coated coronary stent. In a 2015 trial comparing a biolimus A9-coated stent vs a bare-metal stent, patients received DAPT for 1 month after stent placement. The drug-coated stent was found to be superior in terms of the primary safety end point (cardiac death, myocardial infarction, or stent thrombosis).12 This stent is not yet approved by the US Food and Drug Administration at the time of this writing.

Further study is needed to evaluate DAPT durations of less than 3 months and to establish the proper timing for safely discontinuing DAPT in difficult clinical scenarios.

WHEN STOPPING MAY BE REASONABLE

According to current guidelines, in patients at high bleeding risk with a second-generation or newer drug-eluting stent for stable ischemic heart disease, discontinuing DAPT 3 months after stent placement may be reasonable.1 The decision to stop DAPT in these patients requires a careful assessment of the risks and benefits and may be aided by a tool such as the DAPT risk score. However, these recommendations cannot be extrapolated to patients with an acute coronary syndrome within the past year, as they are at higher risk.

TAKE-HOME MESSAGES

  • A cardiologist should be consulted before discontinuing DAPT in patients with a drug-eluting stent, especially if the stent was recently placed.
  • The duration of therapy depends on the indication for stent placement (stable ischemic heart disease vs acute coronary syndrome) and on stent location.
  • Based on the 2016 American College of Cardiology/American Heart Association guidelines,1 in patients at high bleeding risk with a second-generation drug-eluting stent, discontinuing DAPT is safe after 3 months in patients with stable ischemic heart disease, and after 6 months in patients with an acute coronary syndrome.
  • When prescribing DAPT, available evidence favors clopidogrel in patients with stable ischemic heart disease who have a second-generation drug-eluting stent and are at high bleeding risk.
  • In these patients, the risk-benefit ratio based on the DAPT score may be useful when considering stopping clopidogrel.       
References
  1. Levine GN, Bates ER, Bittl JA, et al. 2016 ACC/AHA guideline focused update on duration of dual antiplatelet therapy in patients with coronary artery disease: a report of the American College of Cardiology/American Heart Association Task Force on Clinical Practice Guidelines. Circulation 2016; 134(10):e123–e155. doi:10.1161/CIR.0000000000000404 [correction in doi:10.1161/CIR.0000000000000452]
  2. Mauri L, Kereiakes DJ, Yeh RW, et al; DAPT Study Investigators. Twelve or 30 months of dual antiplatelet therapy after drug-eluting stents. N Engl J Med 2014; 371(23):2155–2166. doi:10.1056/NEJMoa1409312
  3. Bonaca MP, Bhatt DL, Cohen M, et al; PEGASUS-TIMI 54 Steering Committee and Investigators. Long-term use of ticagrelor in patients with prior myocardial infarction. N Engl J Med 2015; 372(19):1791–1800. doi:10.1056/NEJMoa1500857
  4. Bittl JA, Baber U, Bradley SM, Wijeysundera DN. Duration of dual antiplatelet therapy: a systematic review for the 2016 ACC/AHA guideline focused update on duration of dual antiplatelet therapy in patients with coronary artery disease: a report of the American College of Cardiology/American Heart Association Task Force on Clinical Practice Guidelines. J Am Coll Cardiol 2016; 68(10):1116–1139. doi:10.1016/j.jacc.2016.03.512
  5. Feres F, Costa RA, Abizaid A, et al; OPTIMIZE Trial Investigators. Three vs twelve months of dual antiplatelet therapy after zotarolimus-eluting stents: the OPTIMIZE randomized trial. JAMA 2013; 310(23):2510–2522. doi:10.1001/jama.2013.282183
  6. Kubo T, Akasaka T, Kozuma K, et al. Comparison of neointimal coverage between everolimus-eluting stents and sirolimus-eluting stents: an optical coherence tomography substudy of RESET. EuroIntervention 2015. doi:10.4244/EIJV11I5A109
  7. Schulz-Schupke S, Byrne RA, ten Berg JM, et al; Intracoronary Stenting and Antithrombotic Regimen: Safety And EFficacy of 6 Months Dual Antiplatelet Therapy After Drug-Eluting Stenting (ISAR-SAFE) Trial Investigators. ISAR-SAFE: a randomized, double-blind, placebo-controlled trial of 6 vs 12 months of clopidogrel therapy after drug-eluting stenting. Eur Heart J 2015; 36(20):1252–1263. doi:10.1093/eurheartj/ehu523
  8. Gwon HC, Hahn JY, Park KW, et al. Six-month versus 12-month dual antiplatelet therapy after implantation of drug-eluting stents: the efficacy of Xience/Promus vs Cypher to reduce late loss after stenting (EXCELLENT) randomized, multicenter study. Circulation 2012; 125(3):505–513. doi:10.1161/CIRCULATIONAHA.111.059022
  9. Colombo A, Chieffo A, Frasheri A, et al. Second-generation drug-eluting stent implantation followed by 6- vs 12-month dual antiplatelet therapy: the SECURITY randomized clinical trial. J Am Coll Cardiol 2014; 64(20):2086–2097. doi:10.1016/j.jacc.2014.09.008
  10. Kim BK, Hong MK, Shin DH, et al; RESET Investigators. A new strategy for discontinuation of dual antiplatelet therapy: the RESET Trial (REal Safety and Efficacy of 3-month dual antiplatelet Therapy following Endeavor zotarolimus-eluting stent implantation). J Am Coll Cardiol 2012; 60(15):1340–1348. doi:10.1016/j.jacc.2012.06.043
  11. US National Library of Medicine. ClinicalTrials.gov. EVOLVE Short DAPT Study. https://clinicaltrials.gov/ct2/show/NCT02605447. Accessed December 3, 2018.
  12. Urban P, Meredith IT, Abizaid A, et al; LEADERS FREE Investigators. Polymer-free drug-coated coronary stents in patients at high bleeding risk. N Engl J Med 2015; 373(21):2038–2047. doi:10.1056/NEJMoa1503943
Article PDF
Author and Disclosure Information

Juan Simon Rico-Mesa, MD
Department of Medicine, University of Texas Health, San Antonio, TX; Department of Cardiovascular Medicine, Mayo Clinic, Rochester, MN

Carlos Uribe, MD, FACC, FSCAI
Interventional Cardiologist, Associate Professor of Medicine, CES University; Program director of Interventional Cardiology, UPB University, Clinica CardioVID, Hospital Pablo Tobon Uribe, Medellin, Colombia

Megha Prasad, MD
Division of Cardiology, Columbia University Medical Center, New York, NY 

Sushil Allen Luis, MBBS, FRACP
Department of Cardiovascular Medicine, Mayo Clinic, Rochester, MN

Address: Juan Simon Rico-Mesa, MD, Department of Cardiovascular Medicine, Mayo Clinic, 200 First Street SW, Rochester, MN 55905; juansimonrico@hotmail.com

Issue
Cleveland Clinic Journal of Medicine - 86(1)
Publications
Topics
Page Number
13-16
Legacy Keywords
dual antiplatelet therapy, DAPT, stent, drug-eluting stent, DES, aspirin, clopidogrel, Plavix, coronary artery disease, thrombosis, bleeding risk, Juan Simon Rico-Mesa, Carlos Uribe, Megha Prasad, Sushil Allen Luis
Sections
Author and Disclosure Information

Juan Simon Rico-Mesa, MD
Department of Medicine, University of Texas Health, San Antonio, TX; Department of Cardiovascular Medicine, Mayo Clinic, Rochester, MN

Carlos Uribe, MD, FACC, FSCAI
Interventional Cardiologist, Associate Professor of Medicine, CES University; Program director of Interventional Cardiology, UPB University, Clinica CardioVID, Hospital Pablo Tobon Uribe, Medellin, Colombia

Megha Prasad, MD
Division of Cardiology, Columbia University Medical Center, New York, NY 

Sushil Allen Luis, MBBS, FRACP
Department of Cardiovascular Medicine, Mayo Clinic, Rochester, MN

Address: Juan Simon Rico-Mesa, MD, Department of Cardiovascular Medicine, Mayo Clinic, 200 First Street SW, Rochester, MN 55905; juansimonrico@hotmail.com

Author and Disclosure Information

Juan Simon Rico-Mesa, MD
Department of Medicine, University of Texas Health, San Antonio, TX; Department of Cardiovascular Medicine, Mayo Clinic, Rochester, MN

Carlos Uribe, MD, FACC, FSCAI
Interventional Cardiologist, Associate Professor of Medicine, CES University; Program director of Interventional Cardiology, UPB University, Clinica CardioVID, Hospital Pablo Tobon Uribe, Medellin, Colombia

Megha Prasad, MD
Division of Cardiology, Columbia University Medical Center, New York, NY 

Sushil Allen Luis, MBBS, FRACP
Department of Cardiovascular Medicine, Mayo Clinic, Rochester, MN

Address: Juan Simon Rico-Mesa, MD, Department of Cardiovascular Medicine, Mayo Clinic, 200 First Street SW, Rochester, MN 55905; juansimonrico@hotmail.com

Article PDF
Article PDF
Related Articles

Stopping dual antiplatelet therapy (DAPT) (eg, clopidogrel plus aspirin) after 3 months is reasonable in patients with stable ischemic heart disease who have a second-generation drug-eluting stent and a high bleeding risk, with stable ischemic disease defined as at least 1 year free of acute coronary syndromes. However, these patients should continue lifelong aspirin monotherapy. Current guidelines suggest that in stable ischemic disease, the risk-benefit ratio may favor an even shorter duration of DAPT than the 6 months currently recommended.1

STABLE ISCHEMIC HEART DISEASE VS ACUTE CORONARY SYNDROME

Percutaneous coronary intervention for stable ischemic heart disease is indicated primarily in patients with angina that persists despite optimal antianginal therapy.

The prognostic implications of DAPT are different in stable ischemic disease than in acute coronary syndromes. The substrate treated by percutaneous intervention in stable ischemic disease is primarily fibrofatty plaque, as opposed to thrombus in acute coronary syndromes.

Percutaneous intervention significantly improves the prognosis in acute coronary syndromes, whereas its impact on overall survival in stable ischemic heart disease is not well documented. Given these differences, our discussion about DAPT in stable ischemic disease cannot be extrapolated to acute coronary syndromes.

BENEFITS OF DAPT

DAPT is mandatory early after drug-eluting stent placement, when the stent continuously releases medication, inhibiting tissue growth within the lumen of the stent.

Endothelialization of the stent normally occurs during the first 7 to 30 days after placement. During this period, the nonendothelialized stent poses a risk of thrombosis, a life-threatening, catastrophic condition with a mortality rate between 9% and 45%.1

Aspirin 75 to 100 mg has been shown to be effective as secondary prevention of atherosclerotic disease and is recommended lifelong in this clinical setting. Adding a thienopyridine reduces the risk of myocardial infarction, stent thrombosis, and death from a cardiovascular event and decreases the incidence of plaque rupture in nonstented coronary vessels. Hence, prevention of these complications provides the rationale for DAPT in this clinical setting.

THERAPY BEYOND 12 MONTHS

Although guidelines have traditionally recommended 12 months of DAPT, the optimal duration is still debated.

A duration beyond 12 months in patients with a history of myocardial infarction was shown to be reasonable in 2 large trials,2,3 while a 2016 review by Bittl et al4 suggested that therapy beyond 12 months in patients with a newer-generation drug-eluting stent could increase the incidence of major bleeding. A detailed discussion of DAPT longer than 12 months is beyond the scope of this article.

EVIDENCE FOR SHORTER DURATION

The results of 5 major trials support shorter duration of DAPT in stable ischemic disease.

The OPTIMIZE5 and RESET6 trials found that 3 months of DAPT was not inferior to 12 months in terms of ischemic and safety end points.

The ISAR-SAFE,7 EXCELLENT,8 and SECURITY9 trials also reported that 6 months of DAPT was not inferior to 12 months for the primary composite end point of death, stent thrombosis, myocardial infarction, stroke, or major bleeding.

However, these trials may have been underpowered to detect a difference in rates of stent thrombosis with shorter-duration DAPT.

CURRENT GUIDELINES

For patients at high bleeding risk, the current guidelines of the American College of Cardiology and American Heart Association, updated in 2016, suggest that it may be reasonable to discontinue DAPT 3 months after drug-eluting stent placement in patients with stable ischemic heart disease, and at 6 months in patients with acute coronary syndrome (class IIb recommendation, level of evidence C).1 These recommendations are based on results of randomized controlled trials showing no difference in the rate of stent thrombosis and composite ischemic events with a shorter duration than with 12 months of therapy.5–10

The evidence for DAPT in stable ischemic disease is based on clopidogrel, with only limited data on ticagrelor.1 To our knowledge, no study to date has evaluated DAPT in this setting for less than 3 months, and further study is needed to address shorter-duration approaches with current-generation drug-eluting stents Since 2017, all coronary stents implanted in the United States have been second-generation stents.

 

 

TOOLS TO HELP DECISION-MAKING

The decision to stop DAPT in a patient at high risk of bleeding requires a careful assessment of the risks and benefits. Risk factors for bleeding include advanced age, history of major bleeding, anticoagulation, chronic kidney disease (serum creatinine level ≥ 2 mg/dL), platelet count 100 × 109/L or lower, and history of stroke.11

Table 1. Risk factors for ischemia, stent thrombosis, and bleeding
A useful approach is to define the risks of stent thrombosis and bleeding (Table 1).1 The DAPT score determines the risk-benefit ratio for long-term DAPT as follows:

  • Age 75 or older: −2 points
  • Ages 65 to 74: −1
  • Age under 65: 0
  • Diabetes mellitus: 1
  • Myocardial infarction at presentation: 1
  • History of percutaneous coronary intervention or myocardial infarction: 1
  • Stent diameter less than 3 mm: 1
  • Paclitaxel drug-eluting stent: 1
  • Current smoker: 2
  • Percutaneous coronary intervention with saphenous vein graft: 2
  • Congestive heart failure or left ventricular ejection fraction less than 30%: 2.

A score of 2 or greater favors continuing DAPT, as it indicates higher ischemic risk. A score less than 2 favors discontinuing DAPT, as it indicates higher bleeding risk.1,2

IF BLEEDING RISK IS HIGH

Preventing and controlling bleeding associated with DAPT is important. The gastrointestinal tract is the most common site of bleeding.

Aspirin inhibits prostaglandin synthesis, leading to disruption of the protective mucous membrane. Therefore, a proton pump inhibitor should be started along with DAPT in patients at high risk of gastrointestinal bleeding.

If a patient’s bleeding risk significantly outweighs the risk of stent thrombosis, or if active hemorrhage makes a patient hemodynamically unstable, antiplatelet therapy must be stopped.1

FACING SURGERY

For patients with a drug-eluting stent who are on DAPT and are to undergo elective noncardiac surgery, 3 considerations must be kept in mind:

  • The risk of stent thrombosis if DAPT needs to be interrupted
  • The consequences of delaying the surgical procedure
  • The risk and consequences of periprocedural and intraprocedural bleeding if DAPT is continued.

Because clinical evidence for bridging therapy with intravenous antiplatelet or anticoagulant agents is limited, it is difficult to make recommendations about stopping DAPT. However, once bleeding risk is stabilized, DAPT should be restarted as soon as possible.1

CURRENT RESEARCH

Several trials are under way to further evaluate ways to minimize bleeding risk and shorten the duration of DAPT.

A prospective multicenter trial is evaluating 3-month DAPT in patients at high bleeding risk who undergo placement of an everolimus-eluting stent.11 This study is expected to be completed in August 2019.

Another strategy for patients at high bleeding risk is use of a polymer-free drug-coated coronary stent. In a 2015 trial comparing a biolimus A9-coated stent vs a bare-metal stent, patients received DAPT for 1 month after stent placement. The drug-coated stent was found to be superior in terms of the primary safety end point (cardiac death, myocardial infarction, or stent thrombosis).12 This stent is not yet approved by the US Food and Drug Administration at the time of this writing.

Further study is needed to evaluate DAPT durations of less than 3 months and to establish the proper timing for safely discontinuing DAPT in difficult clinical scenarios.

WHEN STOPPING MAY BE REASONABLE

According to current guidelines, in patients at high bleeding risk with a second-generation or newer drug-eluting stent for stable ischemic heart disease, discontinuing DAPT 3 months after stent placement may be reasonable.1 The decision to stop DAPT in these patients requires a careful assessment of the risks and benefits and may be aided by a tool such as the DAPT risk score. However, these recommendations cannot be extrapolated to patients with an acute coronary syndrome within the past year, as they are at higher risk.

TAKE-HOME MESSAGES

  • A cardiologist should be consulted before discontinuing DAPT in patients with a drug-eluting stent, especially if the stent was recently placed.
  • The duration of therapy depends on the indication for stent placement (stable ischemic heart disease vs acute coronary syndrome) and on stent location.
  • Based on the 2016 American College of Cardiology/American Heart Association guidelines,1 in patients at high bleeding risk with a second-generation drug-eluting stent, discontinuing DAPT is safe after 3 months in patients with stable ischemic heart disease, and after 6 months in patients with an acute coronary syndrome.
  • When prescribing DAPT, available evidence favors clopidogrel in patients with stable ischemic heart disease who have a second-generation drug-eluting stent and are at high bleeding risk.
  • In these patients, the risk-benefit ratio based on the DAPT score may be useful when considering stopping clopidogrel.       

Stopping dual antiplatelet therapy (DAPT) (eg, clopidogrel plus aspirin) after 3 months is reasonable in patients with stable ischemic heart disease who have a second-generation drug-eluting stent and a high bleeding risk, with stable ischemic disease defined as at least 1 year free of acute coronary syndromes. However, these patients should continue lifelong aspirin monotherapy. Current guidelines suggest that in stable ischemic disease, the risk-benefit ratio may favor an even shorter duration of DAPT than the 6 months currently recommended.1

STABLE ISCHEMIC HEART DISEASE VS ACUTE CORONARY SYNDROME

Percutaneous coronary intervention for stable ischemic heart disease is indicated primarily in patients with angina that persists despite optimal antianginal therapy.

The prognostic implications of DAPT are different in stable ischemic disease than in acute coronary syndromes. The substrate treated by percutaneous intervention in stable ischemic disease is primarily fibrofatty plaque, as opposed to thrombus in acute coronary syndromes.

Percutaneous intervention significantly improves the prognosis in acute coronary syndromes, whereas its impact on overall survival in stable ischemic heart disease is not well documented. Given these differences, our discussion about DAPT in stable ischemic disease cannot be extrapolated to acute coronary syndromes.

BENEFITS OF DAPT

DAPT is mandatory early after drug-eluting stent placement, when the stent continuously releases medication, inhibiting tissue growth within the lumen of the stent.

Endothelialization of the stent normally occurs during the first 7 to 30 days after placement. During this period, the nonendothelialized stent poses a risk of thrombosis, a life-threatening, catastrophic condition with a mortality rate between 9% and 45%.1

Aspirin 75 to 100 mg has been shown to be effective as secondary prevention of atherosclerotic disease and is recommended lifelong in this clinical setting. Adding a thienopyridine reduces the risk of myocardial infarction, stent thrombosis, and death from a cardiovascular event and decreases the incidence of plaque rupture in nonstented coronary vessels. Hence, prevention of these complications provides the rationale for DAPT in this clinical setting.

THERAPY BEYOND 12 MONTHS

Although guidelines have traditionally recommended 12 months of DAPT, the optimal duration is still debated.

A duration beyond 12 months in patients with a history of myocardial infarction was shown to be reasonable in 2 large trials,2,3 while a 2016 review by Bittl et al4 suggested that therapy beyond 12 months in patients with a newer-generation drug-eluting stent could increase the incidence of major bleeding. A detailed discussion of DAPT longer than 12 months is beyond the scope of this article.

EVIDENCE FOR SHORTER DURATION

The results of 5 major trials support shorter duration of DAPT in stable ischemic disease.

The OPTIMIZE5 and RESET6 trials found that 3 months of DAPT was not inferior to 12 months in terms of ischemic and safety end points.

The ISAR-SAFE,7 EXCELLENT,8 and SECURITY9 trials also reported that 6 months of DAPT was not inferior to 12 months for the primary composite end point of death, stent thrombosis, myocardial infarction, stroke, or major bleeding.

However, these trials may have been underpowered to detect a difference in rates of stent thrombosis with shorter-duration DAPT.

CURRENT GUIDELINES

For patients at high bleeding risk, the current guidelines of the American College of Cardiology and American Heart Association, updated in 2016, suggest that it may be reasonable to discontinue DAPT 3 months after drug-eluting stent placement in patients with stable ischemic heart disease, and at 6 months in patients with acute coronary syndrome (class IIb recommendation, level of evidence C).1 These recommendations are based on results of randomized controlled trials showing no difference in the rate of stent thrombosis and composite ischemic events with a shorter duration than with 12 months of therapy.5–10

The evidence for DAPT in stable ischemic disease is based on clopidogrel, with only limited data on ticagrelor.1 To our knowledge, no study to date has evaluated DAPT in this setting for less than 3 months, and further study is needed to address shorter-duration approaches with current-generation drug-eluting stents Since 2017, all coronary stents implanted in the United States have been second-generation stents.

 

 

TOOLS TO HELP DECISION-MAKING

The decision to stop DAPT in a patient at high risk of bleeding requires a careful assessment of the risks and benefits. Risk factors for bleeding include advanced age, history of major bleeding, anticoagulation, chronic kidney disease (serum creatinine level ≥ 2 mg/dL), platelet count 100 × 109/L or lower, and history of stroke.11

Table 1. Risk factors for ischemia, stent thrombosis, and bleeding
A useful approach is to define the risks of stent thrombosis and bleeding (Table 1).1 The DAPT score determines the risk-benefit ratio for long-term DAPT as follows:

  • Age 75 or older: −2 points
  • Ages 65 to 74: −1
  • Age under 65: 0
  • Diabetes mellitus: 1
  • Myocardial infarction at presentation: 1
  • History of percutaneous coronary intervention or myocardial infarction: 1
  • Stent diameter less than 3 mm: 1
  • Paclitaxel drug-eluting stent: 1
  • Current smoker: 2
  • Percutaneous coronary intervention with saphenous vein graft: 2
  • Congestive heart failure or left ventricular ejection fraction less than 30%: 2.

A score of 2 or greater favors continuing DAPT, as it indicates higher ischemic risk. A score less than 2 favors discontinuing DAPT, as it indicates higher bleeding risk.1,2

IF BLEEDING RISK IS HIGH

Preventing and controlling bleeding associated with DAPT is important. The gastrointestinal tract is the most common site of bleeding.

Aspirin inhibits prostaglandin synthesis, leading to disruption of the protective mucous membrane. Therefore, a proton pump inhibitor should be started along with DAPT in patients at high risk of gastrointestinal bleeding.

If a patient’s bleeding risk significantly outweighs the risk of stent thrombosis, or if active hemorrhage makes a patient hemodynamically unstable, antiplatelet therapy must be stopped.1

FACING SURGERY

For patients with a drug-eluting stent who are on DAPT and are to undergo elective noncardiac surgery, 3 considerations must be kept in mind:

  • The risk of stent thrombosis if DAPT needs to be interrupted
  • The consequences of delaying the surgical procedure
  • The risk and consequences of periprocedural and intraprocedural bleeding if DAPT is continued.

Because clinical evidence for bridging therapy with intravenous antiplatelet or anticoagulant agents is limited, it is difficult to make recommendations about stopping DAPT. However, once bleeding risk is stabilized, DAPT should be restarted as soon as possible.1

CURRENT RESEARCH

Several trials are under way to further evaluate ways to minimize bleeding risk and shorten the duration of DAPT.

A prospective multicenter trial is evaluating 3-month DAPT in patients at high bleeding risk who undergo placement of an everolimus-eluting stent.11 This study is expected to be completed in August 2019.

Another strategy for patients at high bleeding risk is use of a polymer-free drug-coated coronary stent. In a 2015 trial comparing a biolimus A9-coated stent vs a bare-metal stent, patients received DAPT for 1 month after stent placement. The drug-coated stent was found to be superior in terms of the primary safety end point (cardiac death, myocardial infarction, or stent thrombosis).12 This stent is not yet approved by the US Food and Drug Administration at the time of this writing.

Further study is needed to evaluate DAPT durations of less than 3 months and to establish the proper timing for safely discontinuing DAPT in difficult clinical scenarios.

WHEN STOPPING MAY BE REASONABLE

According to current guidelines, in patients at high bleeding risk with a second-generation or newer drug-eluting stent for stable ischemic heart disease, discontinuing DAPT 3 months after stent placement may be reasonable.1 The decision to stop DAPT in these patients requires a careful assessment of the risks and benefits and may be aided by a tool such as the DAPT risk score. However, these recommendations cannot be extrapolated to patients with an acute coronary syndrome within the past year, as they are at higher risk.

TAKE-HOME MESSAGES

  • A cardiologist should be consulted before discontinuing DAPT in patients with a drug-eluting stent, especially if the stent was recently placed.
  • The duration of therapy depends on the indication for stent placement (stable ischemic heart disease vs acute coronary syndrome) and on stent location.
  • Based on the 2016 American College of Cardiology/American Heart Association guidelines,1 in patients at high bleeding risk with a second-generation drug-eluting stent, discontinuing DAPT is safe after 3 months in patients with stable ischemic heart disease, and after 6 months in patients with an acute coronary syndrome.
  • When prescribing DAPT, available evidence favors clopidogrel in patients with stable ischemic heart disease who have a second-generation drug-eluting stent and are at high bleeding risk.
  • In these patients, the risk-benefit ratio based on the DAPT score may be useful when considering stopping clopidogrel.       
References
  1. Levine GN, Bates ER, Bittl JA, et al. 2016 ACC/AHA guideline focused update on duration of dual antiplatelet therapy in patients with coronary artery disease: a report of the American College of Cardiology/American Heart Association Task Force on Clinical Practice Guidelines. Circulation 2016; 134(10):e123–e155. doi:10.1161/CIR.0000000000000404 [correction in doi:10.1161/CIR.0000000000000452]
  2. Mauri L, Kereiakes DJ, Yeh RW, et al; DAPT Study Investigators. Twelve or 30 months of dual antiplatelet therapy after drug-eluting stents. N Engl J Med 2014; 371(23):2155–2166. doi:10.1056/NEJMoa1409312
  3. Bonaca MP, Bhatt DL, Cohen M, et al; PEGASUS-TIMI 54 Steering Committee and Investigators. Long-term use of ticagrelor in patients with prior myocardial infarction. N Engl J Med 2015; 372(19):1791–1800. doi:10.1056/NEJMoa1500857
  4. Bittl JA, Baber U, Bradley SM, Wijeysundera DN. Duration of dual antiplatelet therapy: a systematic review for the 2016 ACC/AHA guideline focused update on duration of dual antiplatelet therapy in patients with coronary artery disease: a report of the American College of Cardiology/American Heart Association Task Force on Clinical Practice Guidelines. J Am Coll Cardiol 2016; 68(10):1116–1139. doi:10.1016/j.jacc.2016.03.512
  5. Feres F, Costa RA, Abizaid A, et al; OPTIMIZE Trial Investigators. Three vs twelve months of dual antiplatelet therapy after zotarolimus-eluting stents: the OPTIMIZE randomized trial. JAMA 2013; 310(23):2510–2522. doi:10.1001/jama.2013.282183
  6. Kubo T, Akasaka T, Kozuma K, et al. Comparison of neointimal coverage between everolimus-eluting stents and sirolimus-eluting stents: an optical coherence tomography substudy of RESET. EuroIntervention 2015. doi:10.4244/EIJV11I5A109
  7. Schulz-Schupke S, Byrne RA, ten Berg JM, et al; Intracoronary Stenting and Antithrombotic Regimen: Safety And EFficacy of 6 Months Dual Antiplatelet Therapy After Drug-Eluting Stenting (ISAR-SAFE) Trial Investigators. ISAR-SAFE: a randomized, double-blind, placebo-controlled trial of 6 vs 12 months of clopidogrel therapy after drug-eluting stenting. Eur Heart J 2015; 36(20):1252–1263. doi:10.1093/eurheartj/ehu523
  8. Gwon HC, Hahn JY, Park KW, et al. Six-month versus 12-month dual antiplatelet therapy after implantation of drug-eluting stents: the efficacy of Xience/Promus vs Cypher to reduce late loss after stenting (EXCELLENT) randomized, multicenter study. Circulation 2012; 125(3):505–513. doi:10.1161/CIRCULATIONAHA.111.059022
  9. Colombo A, Chieffo A, Frasheri A, et al. Second-generation drug-eluting stent implantation followed by 6- vs 12-month dual antiplatelet therapy: the SECURITY randomized clinical trial. J Am Coll Cardiol 2014; 64(20):2086–2097. doi:10.1016/j.jacc.2014.09.008
  10. Kim BK, Hong MK, Shin DH, et al; RESET Investigators. A new strategy for discontinuation of dual antiplatelet therapy: the RESET Trial (REal Safety and Efficacy of 3-month dual antiplatelet Therapy following Endeavor zotarolimus-eluting stent implantation). J Am Coll Cardiol 2012; 60(15):1340–1348. doi:10.1016/j.jacc.2012.06.043
  11. US National Library of Medicine. ClinicalTrials.gov. EVOLVE Short DAPT Study. https://clinicaltrials.gov/ct2/show/NCT02605447. Accessed December 3, 2018.
  12. Urban P, Meredith IT, Abizaid A, et al; LEADERS FREE Investigators. Polymer-free drug-coated coronary stents in patients at high bleeding risk. N Engl J Med 2015; 373(21):2038–2047. doi:10.1056/NEJMoa1503943
References
  1. Levine GN, Bates ER, Bittl JA, et al. 2016 ACC/AHA guideline focused update on duration of dual antiplatelet therapy in patients with coronary artery disease: a report of the American College of Cardiology/American Heart Association Task Force on Clinical Practice Guidelines. Circulation 2016; 134(10):e123–e155. doi:10.1161/CIR.0000000000000404 [correction in doi:10.1161/CIR.0000000000000452]
  2. Mauri L, Kereiakes DJ, Yeh RW, et al; DAPT Study Investigators. Twelve or 30 months of dual antiplatelet therapy after drug-eluting stents. N Engl J Med 2014; 371(23):2155–2166. doi:10.1056/NEJMoa1409312
  3. Bonaca MP, Bhatt DL, Cohen M, et al; PEGASUS-TIMI 54 Steering Committee and Investigators. Long-term use of ticagrelor in patients with prior myocardial infarction. N Engl J Med 2015; 372(19):1791–1800. doi:10.1056/NEJMoa1500857
  4. Bittl JA, Baber U, Bradley SM, Wijeysundera DN. Duration of dual antiplatelet therapy: a systematic review for the 2016 ACC/AHA guideline focused update on duration of dual antiplatelet therapy in patients with coronary artery disease: a report of the American College of Cardiology/American Heart Association Task Force on Clinical Practice Guidelines. J Am Coll Cardiol 2016; 68(10):1116–1139. doi:10.1016/j.jacc.2016.03.512
  5. Feres F, Costa RA, Abizaid A, et al; OPTIMIZE Trial Investigators. Three vs twelve months of dual antiplatelet therapy after zotarolimus-eluting stents: the OPTIMIZE randomized trial. JAMA 2013; 310(23):2510–2522. doi:10.1001/jama.2013.282183
  6. Kubo T, Akasaka T, Kozuma K, et al. Comparison of neointimal coverage between everolimus-eluting stents and sirolimus-eluting stents: an optical coherence tomography substudy of RESET. EuroIntervention 2015. doi:10.4244/EIJV11I5A109
  7. Schulz-Schupke S, Byrne RA, ten Berg JM, et al; Intracoronary Stenting and Antithrombotic Regimen: Safety And EFficacy of 6 Months Dual Antiplatelet Therapy After Drug-Eluting Stenting (ISAR-SAFE) Trial Investigators. ISAR-SAFE: a randomized, double-blind, placebo-controlled trial of 6 vs 12 months of clopidogrel therapy after drug-eluting stenting. Eur Heart J 2015; 36(20):1252–1263. doi:10.1093/eurheartj/ehu523
  8. Gwon HC, Hahn JY, Park KW, et al. Six-month versus 12-month dual antiplatelet therapy after implantation of drug-eluting stents: the efficacy of Xience/Promus vs Cypher to reduce late loss after stenting (EXCELLENT) randomized, multicenter study. Circulation 2012; 125(3):505–513. doi:10.1161/CIRCULATIONAHA.111.059022
  9. Colombo A, Chieffo A, Frasheri A, et al. Second-generation drug-eluting stent implantation followed by 6- vs 12-month dual antiplatelet therapy: the SECURITY randomized clinical trial. J Am Coll Cardiol 2014; 64(20):2086–2097. doi:10.1016/j.jacc.2014.09.008
  10. Kim BK, Hong MK, Shin DH, et al; RESET Investigators. A new strategy for discontinuation of dual antiplatelet therapy: the RESET Trial (REal Safety and Efficacy of 3-month dual antiplatelet Therapy following Endeavor zotarolimus-eluting stent implantation). J Am Coll Cardiol 2012; 60(15):1340–1348. doi:10.1016/j.jacc.2012.06.043
  11. US National Library of Medicine. ClinicalTrials.gov. EVOLVE Short DAPT Study. https://clinicaltrials.gov/ct2/show/NCT02605447. Accessed December 3, 2018.
  12. Urban P, Meredith IT, Abizaid A, et al; LEADERS FREE Investigators. Polymer-free drug-coated coronary stents in patients at high bleeding risk. N Engl J Med 2015; 373(21):2038–2047. doi:10.1056/NEJMoa1503943
Issue
Cleveland Clinic Journal of Medicine - 86(1)
Issue
Cleveland Clinic Journal of Medicine - 86(1)
Page Number
13-16
Page Number
13-16
Publications
Publications
Topics
Article Type
Display Headline
When can I stop dual antiplatelet therapy in patients with drug-eluting stents?
Display Headline
When can I stop dual antiplatelet therapy in patients with drug-eluting stents?
Legacy Keywords
dual antiplatelet therapy, DAPT, stent, drug-eluting stent, DES, aspirin, clopidogrel, Plavix, coronary artery disease, thrombosis, bleeding risk, Juan Simon Rico-Mesa, Carlos Uribe, Megha Prasad, Sushil Allen Luis
Legacy Keywords
dual antiplatelet therapy, DAPT, stent, drug-eluting stent, DES, aspirin, clopidogrel, Plavix, coronary artery disease, thrombosis, bleeding risk, Juan Simon Rico-Mesa, Carlos Uribe, Megha Prasad, Sushil Allen Luis
Sections
Disallow All Ads
Content Gating
No Gating (article Unlocked/Free)
Alternative CME
Disqus Comments
Default
Gate On Date
Wed, 01/02/2019 - 08:45
Un-Gate On Date
Wed, 01/02/2019 - 08:45
Use ProPublica
CFC Schedule Remove Status
Wed, 01/02/2019 - 08:45
Article PDF Media

Shortness of breath, fever, cough, and more in an elderly woman

Article Type
Changed
Tue, 09/03/2019 - 09:01
Display Headline
Shortness of breath, fever, cough, and more in an elderly woman

An 87-year-old woman was brought to the intensive care unit with worsening shortness of breath on exertion, fatigue, orthopnea, paroxysmal nocturnal dyspnea, lower extremity swelling, subjective fever, productive cough, and rhinorrhea over the last week. She reported no chest pain, lightheadedness, or palpitations. Her medical history included the following:

  • Coronary artery disease requiring coronary artery bypass grafting
  • Ischemic cardiomyopathy
  • Severe mitral regurgitation
  • Moderate tricuspid regurgitation
  • Pulmonary hypertension
  • Cardiac arrest with recurrent ventricular tachycardia requiring an implanted cardioverter-defibrillator and amiodarone therapy
  • Hypothyroidism requiring levothyroxine
  • Asthma with a moderate obstructive pattern: forced expiratory volume in 1 second (FEV1) 60% of predicted, forced vital capacity (FVC) 2.06 L, FEV1/FVC 54%, diffusing capacity for carbon monoxide (DLCO) 72% of predicted with positive bronchodilator response
  • Long-standing essential thrombocythemia treated with hydroxyurea.

Before admission, she had been reliably taking guideline-directed heart failure therapy as well as amiodarone for her recurrent ventricular tachycardia. Her levothyroxine had recently been increased as well.

The patient's laboratory values
Physical examination. On admission, her blood pressure was 95/53 mm Hg, heart rate 73 beats per minute, temperature 36.7ºC (98.1ºF), and oxygen saturation 81% requiring supplemental oxygen 15 L/min by nonrebreather face mask. Physical examination revealed elevated jugular venous pressure, bibasilar crackles, lower extremity edema, and a grade 3 of 6 holosystolic murmur both at the left sternal border and at the apex radiating to the axilla. There was no evidence of wheezing or pulsus paradoxus.

Initial laboratory evaluation revealed abnormal values (Table 1).

Electrocardiography showed sinus rhythm and an old left bundle branch block.

Chest radiography showed cardiomegaly, bilateral pleural effusions, and pulmonary edema.

WHAT IS THE CAUSE OF HER SYMPTOMS?

1. Based on the available information, which of the following is the most likely cause of this patient’s clinical presentation?

  • Acute decompensated heart failure
  • Pulmonary embolism
  • Exacerbation of asthma
  • Exacerbation of chronic obstructive pulmonary disease (COPD)

Heart failure is a clinical diagnosis based on careful history-taking and physical examination. Major criteria include paroxysmal nocturnal dyspnea, orthopnea, elevated jugular venous pressure, pulmonary crackles, a third heart sound, cardiomegaly, pulmonary edema, and weight loss of more than 4.5 kg with diuretic therapy.1 N-terminal pro-B-type natriuretic peptide (NT-proBNP) is also an effective marker of acute decompensated heart failure in the proper clinical setting.2

Our patient’s elevated jugular venous pressure, bibasilar crackles, lower extremity edema, chest radiography findings consistent with pulmonary edema, markedly elevated NT-proBNP, history of orthopnea, paroxysmal nocturnal dyspnea, and dyspnea on exertion were most consistent with acute decompensated heart failure. Her cough and subjective fevers were thought to be due to an upper respiratory tract viral infection.

Pulmonary embolism causes pleuritic chest pain, dyspnea, and, occasionally, elevated troponin. The most common feature on electrocardiography is sinus tachycardia; nonspecific ST-segment and T-wave changes may also be seen.3

Although pulmonary embolism remained in the differential diagnosis, our patient’s lack of typical features of pulmonary embolism made this less likely.

Asthma is characterized by recurrent airflow obstruction and bronchial hyperresponsiveness.4 Asthma exacerbations present with wheezing, tachypnea, tachycardia, and pulsus paradoxus.5

Despite her previous asthma diagnosis, our patient’s lack of typical features of asthma exacerbation made this diagnosis unlikely.

COPD exacerbations present with increased dyspnea, cough, sputum production, wheezing, lung resonance to percussion, and distant heart sounds, and are characterized by airflow obstruction.6,7

Although our patient presented with cough and dyspnea, she had no history of COPD and her other signs and symptoms (elevated jugular venous pressure, elevated NT-proBNP, and peripheral edema) could not be explained by COPD exacerbation.

 

 

OUR PATIENT UNDERWENT FURTHER TESTING

Echocardiography revealed severe left ventricular enlargement, an ejection fraction of 20% (which was near her baseline value), diffuse regional wall-motion abnormalities, severe mitral regurgitation, and moderate tricuspid regurgitation consistent with an exacerbation of heart failure.

We considered the possibility that her heart failure symptoms might be due to precipitous up-titration of her levothyroxine dose, given her borderline-elevated free thyroxine (T4) and increase in cardiac index (currently 4.45 L/min/m2, previously 2.20 L/min/m2 by the left ventricular outflow tract velocity time integral method). However, given her reduced ejection fraction, this clinical presentation most likely represented an acute exacerbation of her chronic heart failure. Her subjective fevers were thought to be due to a viral infection of the upper respiratory tract. The macrocytic anemia and thrombocytopenia were thought to be a side effect of her long-standing treatment with hydroxyurea for essential thrombocythemia, although amiodarone has also been associated with cytopenia.8

Treatment was started with intravenous diuretics and positive pressure ventilation with oxygen supplementation. Her levothyroxine dose was reduced, and her hydroxyurea was stopped.

Chest computed tomography axial views
Figure 1. Chest computed tomography axial views demonstrated increased attenuation in the liver (A, arrow) and left pleural base (B, arrow). Evaluation of the right lung base revealed ground-glass opacities (C, arrow) and honeycombing (D, arrow). These findings were consistent with amiodarone pulmonary toxicity.
After aggressive diuresis, our patient returned to euvolemia. However, she had persistent fine crackles and hypoxia. She had no further fever, and her vital signs were otherwise stable. Her cytopenia improved with cessation of hydroxyurea. Chest computed tomography (CT) showed bibasilar ground-glass infiltrates with areas of interstitial fibrosis, high-attenuation pleural lesions, and increased liver attenuation (Figure 1).

Further testing for connective tissue disease and hypersensitivity pneumonitis was also done, and the results were negative. To exclude an atypical infection, bronchoalveolar lavage was performed; preliminary microbial testing was negative, and the white blood cell count in the lavage fluid was 90% macrophages (pigment-laden), 7% neutrophils, and 3% lymphocytes.

WHAT IS THE CAUSE OF HER PERSISTENT PULMONARY FINDINGS?

2. Given the CT findings and laboratory results, what is the most likely cause of our patient’s persistent crackles and hypoxia?

  • Heart failure with reduced ejection fraction
  • Bacterial pneumonia
  • Idiopathic pulmonary fibrosis
  • Amiodarone pulmonary toxicity

Heart failure with reduced ejection fraction can cause ground-glass opacities on CT due to increased pulmonary edema. Although our patient initially presented with acute decompensation of heart failure with reduced ejection fraction decompensation, she had returned to euvolemia after aggressive diuresis. Moreover, increased pleural and liver attenuation are not typically seen as a result of heart failure with reduced ejection fraction, making this diagnosis less likely.

Bacterial pneumonia typically presents with cough, fever, and purulent sputum production.9 Further evaluation usually reveals decreased breath sounds, dullness to percussion, and leukocytosis.10 Chest CT in bacterial pneumonia commonly shows a focal area of consolidation, which was not seen in our patient.11

Idiopathic pulmonary fibrosis usually presents with slowly progressive dyspnea and nonproductive cough.12 Physical examination usually reveals fine crackles and occasionally end-inspiratory “squeaks” if traction bronchiectasis is present.12 The diagnosis of idiopathic pulmonary fibrosis requires chest CT findings compatible with it (ie, basal fibrosis, reticular abnormalities, and honeycombing). However, it remains a diagnosis of exclusion and requires ruling out conditions known to cause pulmonary fibrosis such as hypersensitivity pneumonitis, connective tissue disease, and certain medications.12

Although idiopathic pulmonary fibrosis remained in the differential diagnosis, our patient remained on amiodarone, a known cause of pulmonary fibrosis.13 Similarly, the high-attenuation pleural lesions likely represented organizing pneumonia, which is more common in amiodarone pulmonary toxicity. And the ground-glass opacities made idiopathic pulmonary fibrosis unlikely, although they may be seen in an acute exacerbation of this disease.14 Thus, a diagnosis of idiopathic pulmonary fibrosis could not be made definitively.

Amiodarone pulmonary toxicity most commonly presents with acute to subacute cough and progressive dyspnea.13 Physical findings are similar to those in idiopathic pulmonary fibrosis and commonly include bibasilar crackles. Chest CT shows diffuse ground-glass opacities, reticular abnormalities, fibrosis, and increased attenuation of multiple organs, including the lungs, liver, and spleen.14 Bronchoalveolar lavage findings of lipid-laden macrophages suggest but do not definitively diagnose amiodarone pulmonary toxicity.15 And patients with acute amiodarone pulmonary toxicity may present with pigment-laden macrophages on bronchoalveolar lavage, as in our patient.16

Exclusion of hypersensitivity pneumonitis, connective tissue disease, and infection made our patient’s progressive dyspnea and chest CT findings of ground-glass opacities, fibrosis, and increased pulmonary and liver attenuation most consistent with amiodarone pulmonary toxicity.

Amiodarone was therefore discontinued. However, the test result of her lavage fluid for influenza A by polymerase chain reaction came back positive a few hours later.

 

 

WHAT IS THE NEXT STEP?

3. Given the positive influenza A polymerase chain reaction test, which of the following is the best next step in this patient’s management?

  • Surgical lung biopsy
  • Stop amiodarone and start supportive influenza management
  • Stop amiodarone and start dronedarone
  • Start an intravenous corticosteroid

Surgical lung biopsy is typically not required for diagnosis in patients with suspected amiodarone pulmonary toxicity. In addition, acute respiratory distress syndrome has been documented in patients who have undergone surgical biopsy for suspected amiodarone pulmonary toxicity.17

Thus, surgical biopsy is typically only done in cases of persistent symptoms despite withdrawal of amiodarone and initiation of steroid therapy.

Stopping amiodarone and starting supportive influenza management are the best next steps, as our patient’s fevers, cough, dyspnea, and laboratory test results were consistent with influenza.18 Moreover, CT findings of ground-glass opacities and reticular abnormalities can be seen in influenza.19

However, concomitant amiodarone pulmonary toxicity could not be ruled out, as CT showed increased lung and liver attenuation and fibrosis that could not be explained by influenza. And the elevation in aminotransferase levels more than 2 times the upper limit of normal and CT findings of increased liver attenuation suggested amiodarone hepatotoxicity. However, definitive diagnosis would require exclusion of other causes such as congestive hepatopathy, in some cases with liver biopsy.13

Our patient’s persistent hypoxia was thought to be due in part to influenza, and thus the best next step in management was to stop amiodarone and provide supportive care for influenza.

Dronedarone is an antiarrhythmic drug structurally and functionally similar to amiodarone. There are far fewer reports of pulmonary toxicity with dronedarone than with amiodarone.20 However, lack of data on dronedarone in amiodarone pulmonary toxicity, increased rates of hospitalization and death associated with dronedarone in patients like ours with advanced heart failure, and our patient’s previously implanted cardioverter-defibrillator for recurrent ventricular tachycardia all made dronedarone an undesirable alternative to amiodarone.21

Corticosteroids are useful in the treatment of amiodarone pulmonary toxicity when hypoxia and dyspnea are present at diagnosis.13 Our patient’s hypoxia and dyspnea were thought to be due in part to her acute influenza infection, and therefore corticosteroids were not used at the outset.

However, concomitant amiodarone pulmonary toxicity could not be excluded, and the elevation in aminotransferases of more than 2 times the upper limit of normal and CT findings of increased liver attenuation suggested amiodarone hepatotoxicity—though congestive hepatopathy remained in the differential diagnosis. Therefore, supportive therapy for influenza was instituted, and amiodarone was withheld. Her condition subsequently improved, and she was discharged.

FOLLOW-UP 1 MONTH LATER

At a follow-up visit 1 month later, our patient continued to have dyspnea and hypoxia. She did not have signs or symptoms consistent with decompensated heart failure.

Pulmonary function testing revealed the following values:

  • FEV1 0.69 L (56% of predicted)
  • FVC 1.08 L (64% of predicted)
  • Chest computed tomography
    Figure 2. In A, repeat chest computed tomography demonstrated increased liver attenuation (arrow); in B, it showed persistent ground-glass opacities (white arrow), increased pulmonary attenuation (black arrowhead), and worsening pleural effusions (black arrows). These findings supported the diagnosis of amiodarone pulmonary toxicity.
    FEV1/FVC ratio 64%
  • DLCO 2.20 mL/min/mm Hg (12% of predicted).

Aminotransferase levels had also normalized. Repeat chest CT showed persistent bibasilar interstitial fibrotic changes, enlarging bilateral pleural effusions, and persistent peripheral ground-glass opacities (Figure 2).

 

 

WHAT FURTHER TREATMENT IS APPROPRIATE?

4. Given the chest CT findings, which of the following is the most appropriate treatment strategy for this patient?

  • No further management, continue to hold amiodarone
  • Corticosteroids
  • Repeat bronchoalveolar lavage
  • Intravenous antibiotics

No further management of amiodarone pulmonary toxicity would be appropriate if our patient did not have a high burden of symptoms. However, when patients with amiodarone pulmonary toxicity present with hypoxia and dyspnea, corticosteroids should be started.13 Our patient remained symptomatic after discontinuation of amiodarone and resolution of her influenza infection, and CT showed persistent signs of amiodarone pulmonary toxicity, which required further management.

Corticosteroids are useful in treating amiodarone pulmonary toxicity when hypoxia and dyspnea are present at diagnosis. Our patient’s persistent ground-glass opacities, fibrotic changes, and increased attenuation in multiple organs on CT, coupled with a confirmed reduction in FVC of greater than 15% and reduction in DLCO of greater than 20% after recovery from influenza, were most consistent with persistent amiodarone pulmonary toxicity.13

Although our patient’s amiodarone had been discontinued, the long half-life of the drug (45 days) allowed pulmonary toxicity to progress even after the drug was discontinued.22 Because our patient continued to have hypoxia and dyspnea on exertion, the most appropriate next step in management (in addition to managing her pleural effusions) was to start corticosteroids.

For amiodarone pulmonary toxicity, prednisone is typically started at 40 to 60 mg daily and can result in rapid improvement in symptoms.13 Tapering should be slow and may take several months.

Bronchoalveolar lavage is typically used in suspected cases of amiodarone pulmonary toxicity only to rule out an alternative diagnosis such as infection. Lipid-laden macrophages may be seen in the fluid. However, lipid-laden macrophages are not diagnostic of amiodarone pulmonary toxicity, as this finding may also be seen in patients taking amiodarone who do not develop pulmonary toxicity.15 Other findings on bronchoalveolar lavage in amiodarone pulmonary toxicity are nonspecific and are not diagnostically useful.13

Intravenous antibiotics are appropriate if bacterial pneumonia is suspected. However, bacterial pneumonia typically presents with cough, fever, purulent sputum production, and focal consolidation on chest imaging.9 Our patient’s CT findings of persistent peripheral ground-glass opacities and lack of cough, fever, or purulent sputum production were not consistent with bacterial pneumonia, and therefore intravenous antibiotics were not indicated.

CASE CONCLUSION

Given our patient’s persistent dyspnea, hypoxia, and chest CT findings consistent with amiodarone pulmonary toxicity, it was recommended that she start corticosteroids. However, before starting therapy, she suffered a femoral fracture that required surgical intervention. Around the time of the procedure, she had an ST-segment elevation myocardial infarction requiring vasopressor support and mechanical ventilation. At that time, the patient and family decided to pursue comfort measures, and she died peacefully.

MORE ABOUT AMIODARONE PULMONARY TOXICITY

Pulmonary toxicity is a well-described consequence of amiodarone therapy.23 Amiodarone carries a 2% risk of pulmonary toxicity.24 Although higher doses are more likely to cause pulmonary toxicity, lower doses also have been implicated.22,24 Preexisting pulmonary disease may predispose patients taking amiodarone to pulmonary toxicity; however, this is not uniformly seen.25

Mortality rates as high as 10% from amiodarone pulmonary toxicity have been reported. Thus, diligent surveillance for pulmonary toxicity with pulmonary function tests in patients taking amiodarone is mandatory. In particular, a reduction in FVC of greater than 15% or in DLCO of greater than 20% from baseline may be seen in amiodarone pulmonary toxicity.26

Amiodarone pulmonary toxicity can present at any time after the start of therapy, but it occurs most often after 6 to 12 months.13 Patients typically experience insidious dyspnea; however, presentation with acute to subacute cough and progressive dyspnea can occur, especially with high concentrations of supplemental oxygen with or without mechanical ventilation.12,27 Findings on physical examination include bibasilar crackles. CT chest findings include diffuse ground-glass opacities, reticular abnormalities, fibrosis, and increased attenuation in multiple organs, including the lung, liver, and spleen.14

The diagnosis of amiodarone pulmonary toxicity requires ruling out hypersensitivity pneumonitis, connective tissue disease, heart failure, and infection. Surgical biopsy and bronchoalveolar lavage are not commonly used to establish the diagnosis of amiodarone pulmonary toxicity, as surgical biopsy increases the risk of acute respiratory distress syndrome, and the results of bronchoalveolar lavage are usually nonspecific.13,15

Initial treatment involves discontinuing the amiodarone once the diagnosis is suspected. If patients have worsening hypoxia or dyspnea at the time of diagnosis, corticosteroids can be used. Prednisone is typically started at 40 to 60 mg daily and can result in rapid improvement in symptoms.13 Tapering of corticosteroids should occur slowly and may take several months.

References
  1. McKee PA, Castelli WP, McNamara PM, Kannel WB. The natural history of congestive heart failure: the Framingham study. N Engl J Med 1971; 285(26):1441–1446. doi:10.1056/NEJM197112232852601
  2. Stein PD, Terrin ML, Hales CA, et al. Clinical, laboratory, roentgenographic, and electrocardiographic findings in patients with acute pulmonary embolism and no pre-existing cardiac or pulmonary disease. Chest 1991; 100(3):598–603. pmid:1909617
  3. Baggish AL, Siebert U, Lainchbury JG, et al. A validated clinical and biochemical score for the diagnosis of acute heart failure: the ProBNP investigation of dyspnea in the emergency department (PRIDE) acute heart failure score. Am Heart J 2006; 151(1):48–54. doi:10.1016/j.ahj.2005.02.031
  4. National Heart, Lung, and Blood Institute. National Asthma Education and Prevention Program. Expert panel report 3: Guidelines for the diagnosis and management of asthma. www.nhlbi.nih.gov/sites/default/files/media/docs/asthgdln_1.pdf. Accessed August 3, 2018.
  5. Brenner BE, Abraham E, Simon RR. Position and diaphoresis in acute asthma. Am J Med 1983; 74(6):1005–1009. pmid:6407304
  6. Badgett RG, Tanaka DJ, Hunt DK, et al. Can moderate chronic obstructive pulmonary disease be diagnosed by historical and physical findings alone? Am J Med 1993; 94(2):188–196. pmid:8430714
  7. Global Initiative for Chronic Obstructive Lung Disease (GOLD). Global strategy for the diagnosis, management, and prevention of chronic obstructive pulmonary disease. https://goldcopd.org/wp-content/uploads/2017/11/GOLD-2018-v6.0-FINAL-revised-20-Nov_WMS.pdf. Accessed August 17, 2018.
  8. Erie AJ, McClure RF, Wolanskyj AP. Amiodarone-induced bone marrow granulomas: an unusual cause of reversible pancytopenia. Hematol Rep 2010; 2(1):e6. doi:10.4081/hr.2010.e6
  9. Marrie TJ. Community-acquired pneumonia. Clin Infect Dis 1994; 18(4):501–513. pmid:8038304
  10. Metlay JP, Kapoor WN, Fine MJ. Does this patient have community-acquired pneumonia? Diagnosing pneumonia by history and physical examination. JAMA 1997; 278(17):1440–1445. pmid:9356004
  11. Walker CM, Abbott GF, Greene RE, Shepard JA, Vummidi D, Digumarthy SR. Imaging pulmonary infection: classic signs and patterns. AJR Am J Roentgenol 2014; 202(3):479–492. doi:10.2214/AJR.13.11463
  12. Raghu G, Collard HR, Egan JJ, et al; ATS/ERS/JRS/ALAT Committee on Idiopathic Pulmonary Fibrosis. An official ATS/ERS/JRS/ALAT statement: idiopathic pulmonary fibrosis: evidence-based guidelines for diagnosis and management. Am J Respir Crit Care Med 2011; 183(6):788–824. doi:10.1164/rccm.2009-040GL
  13. Goldschlager N, Epstein AE, Naccarelli GV, et al; Practice Guidelines Sub-committee, North American Society of Pacing and Electrophysiology (HRS). A practical guide for clinicians who treat patients with amiodarone: 2007. Heart Rhythm 2007; 4(9):1250–1259. doi:10.1016/j.hrthm.2007.07.020
  14. Kuhlman JE, Teigen C, Ren H, Hruban RH, Hutchins GM, Fishman EK. Amiodarone pulmonary toxicity: CT findings in symptomatic patients. Radiology 1990; 177(1):121–125. doi:10.1148/radiology.177.1.2399310
  15. Martin WJ 2nd, Rosenow EC 3rd. Amiodarone pulmonary toxicity: recognition and pathogenesis (Part I). Chest 1988; 93(5):1067–1075. pmid:3282816
  16. Iskandar SB, Abi-Saleh B, Keith RL, Byrd RP Jr, Roy TM. Amiodarone-induced alveolar hemorrhage. South Med J 2006; 99(4):383–387.
  17. Van Mieghem W, Coolen L, Malysse I, Lacquet LM, Deneffe GJ, Demedts MG. Amiodarone and the development of ARDS after lung surgery. Chest 1994; 105(6):1642–1645. pmid:8205854
  18. Nicholson KG. Clinical features of influenza. Semin Respir Infect 1992; 7(1):26–37. pmid:1609165
  19. Muller NL, Franquet T, Lee KS, Silva CIS. Viruses, mycoplasma, and chlamydia. In: Imaging of Pulmonary Infections. Philadelphia, PA: Lippincott Williams & Wilkins; 2007:94–114.
  20. Stack S, Nguyen DV, Casto A, Ahuja N. Diffuse alveolar damage in a patient receiving dronedarone. Chest 2015; 147(4):e131–e133. doi:10.1378/chest.14-1849
  21. De Ferrari GM, Dusi V. Drug safety evaluation of dronedarone in atrial fibrillation. Expert Opin Drug Saf 2012; 11(6):1023–1045. doi:10.1517/14740338.2012.722994
  22. Okayasu K, Takeda Y, Kojima J, et al. Amiodarone pulmonary toxicity: a patient with three recurrences of pulmonary toxicity and consideration of the probable risk for relapse. Intern Med 2006; 45(22):1303–1307. pmid:17170505
  23. Vorperian VR, Havighurst TC, Miller S, January CT. Adverse effects of low dose amiodarone: a meta-analysis. J Am Coll Cardiol 1997; 30(3):791–798. pmid:9283542
  24. Amiodarone Trials Meta-Analysis Investigators. Effect of prophylactic amiodarone on mortality after acute myocardial infarction and in congestive heart failure: meta-analysis of individual data from 6500 patients in randomised trials. Lancet 1997; 350(9089):1417–1424. pmid:9371164
  25. Olshansky B, Sami M, Rubin A, et al; NHLBI AFFIRM Investigators. Use of amiodarone for atrial fibrillation in patients with preexisting pulmonary disease in the AFFIRM study. Am J Cardiol 2005; 95(3):404–405. doi:10.1016/j.amjcard.2004.09.044
  26. Camus P. Interstitial lung disease from drugs, biologics, and radiation. In: Schwartz MI, King TE Jr, eds. Interstitial Lung Disease. 5th ed. Shelton, CT: People’s Medical Publishing House; 2011:637–644.
  27. Wolkove N, Baltzan M. Amiodarone pulmonary toxicity. Can Respir J 2009; 16(2):43–48. doi:10.1155/2009/282540
Article PDF
Author and Disclosure Information

Brody D. Slostad, MD
Department of Internal Medicine, Mayo Clinic, Rochester, MN

Megha Prasad, MD
Department of Cardiovascular Medicine, Mayo Clinic, Rochester, MN

Nandan S. Anavekar, MD
Department of Cardiovascular Medicine, Mayo Clinic, Rochester, MN

Address: Brody D. Slostad, MD, Department of Internal Medicine, Mayo Clinic, 200 1st Street SW, Rochester, MN 55905; slostad.brody@mayo.edu

Issue
Cleveland Clinic Journal of Medicine - 85(9)
Publications
Topics
Page Number
685-692
Legacy Keywords
amiodarone, pulmonary toxicity, lung toxicity, shortness of breath, heart failure, ventricular tachycardia, Brody Slostad, Megha Prasad, Nandan Anavekar
Sections
Author and Disclosure Information

Brody D. Slostad, MD
Department of Internal Medicine, Mayo Clinic, Rochester, MN

Megha Prasad, MD
Department of Cardiovascular Medicine, Mayo Clinic, Rochester, MN

Nandan S. Anavekar, MD
Department of Cardiovascular Medicine, Mayo Clinic, Rochester, MN

Address: Brody D. Slostad, MD, Department of Internal Medicine, Mayo Clinic, 200 1st Street SW, Rochester, MN 55905; slostad.brody@mayo.edu

Author and Disclosure Information

Brody D. Slostad, MD
Department of Internal Medicine, Mayo Clinic, Rochester, MN

Megha Prasad, MD
Department of Cardiovascular Medicine, Mayo Clinic, Rochester, MN

Nandan S. Anavekar, MD
Department of Cardiovascular Medicine, Mayo Clinic, Rochester, MN

Address: Brody D. Slostad, MD, Department of Internal Medicine, Mayo Clinic, 200 1st Street SW, Rochester, MN 55905; slostad.brody@mayo.edu

Article PDF
Article PDF
Related Articles

An 87-year-old woman was brought to the intensive care unit with worsening shortness of breath on exertion, fatigue, orthopnea, paroxysmal nocturnal dyspnea, lower extremity swelling, subjective fever, productive cough, and rhinorrhea over the last week. She reported no chest pain, lightheadedness, or palpitations. Her medical history included the following:

  • Coronary artery disease requiring coronary artery bypass grafting
  • Ischemic cardiomyopathy
  • Severe mitral regurgitation
  • Moderate tricuspid regurgitation
  • Pulmonary hypertension
  • Cardiac arrest with recurrent ventricular tachycardia requiring an implanted cardioverter-defibrillator and amiodarone therapy
  • Hypothyroidism requiring levothyroxine
  • Asthma with a moderate obstructive pattern: forced expiratory volume in 1 second (FEV1) 60% of predicted, forced vital capacity (FVC) 2.06 L, FEV1/FVC 54%, diffusing capacity for carbon monoxide (DLCO) 72% of predicted with positive bronchodilator response
  • Long-standing essential thrombocythemia treated with hydroxyurea.

Before admission, she had been reliably taking guideline-directed heart failure therapy as well as amiodarone for her recurrent ventricular tachycardia. Her levothyroxine had recently been increased as well.

The patient's laboratory values
Physical examination. On admission, her blood pressure was 95/53 mm Hg, heart rate 73 beats per minute, temperature 36.7ºC (98.1ºF), and oxygen saturation 81% requiring supplemental oxygen 15 L/min by nonrebreather face mask. Physical examination revealed elevated jugular venous pressure, bibasilar crackles, lower extremity edema, and a grade 3 of 6 holosystolic murmur both at the left sternal border and at the apex radiating to the axilla. There was no evidence of wheezing or pulsus paradoxus.

Initial laboratory evaluation revealed abnormal values (Table 1).

Electrocardiography showed sinus rhythm and an old left bundle branch block.

Chest radiography showed cardiomegaly, bilateral pleural effusions, and pulmonary edema.

WHAT IS THE CAUSE OF HER SYMPTOMS?

1. Based on the available information, which of the following is the most likely cause of this patient’s clinical presentation?

  • Acute decompensated heart failure
  • Pulmonary embolism
  • Exacerbation of asthma
  • Exacerbation of chronic obstructive pulmonary disease (COPD)

Heart failure is a clinical diagnosis based on careful history-taking and physical examination. Major criteria include paroxysmal nocturnal dyspnea, orthopnea, elevated jugular venous pressure, pulmonary crackles, a third heart sound, cardiomegaly, pulmonary edema, and weight loss of more than 4.5 kg with diuretic therapy.1 N-terminal pro-B-type natriuretic peptide (NT-proBNP) is also an effective marker of acute decompensated heart failure in the proper clinical setting.2

Our patient’s elevated jugular venous pressure, bibasilar crackles, lower extremity edema, chest radiography findings consistent with pulmonary edema, markedly elevated NT-proBNP, history of orthopnea, paroxysmal nocturnal dyspnea, and dyspnea on exertion were most consistent with acute decompensated heart failure. Her cough and subjective fevers were thought to be due to an upper respiratory tract viral infection.

Pulmonary embolism causes pleuritic chest pain, dyspnea, and, occasionally, elevated troponin. The most common feature on electrocardiography is sinus tachycardia; nonspecific ST-segment and T-wave changes may also be seen.3

Although pulmonary embolism remained in the differential diagnosis, our patient’s lack of typical features of pulmonary embolism made this less likely.

Asthma is characterized by recurrent airflow obstruction and bronchial hyperresponsiveness.4 Asthma exacerbations present with wheezing, tachypnea, tachycardia, and pulsus paradoxus.5

Despite her previous asthma diagnosis, our patient’s lack of typical features of asthma exacerbation made this diagnosis unlikely.

COPD exacerbations present with increased dyspnea, cough, sputum production, wheezing, lung resonance to percussion, and distant heart sounds, and are characterized by airflow obstruction.6,7

Although our patient presented with cough and dyspnea, she had no history of COPD and her other signs and symptoms (elevated jugular venous pressure, elevated NT-proBNP, and peripheral edema) could not be explained by COPD exacerbation.

 

 

OUR PATIENT UNDERWENT FURTHER TESTING

Echocardiography revealed severe left ventricular enlargement, an ejection fraction of 20% (which was near her baseline value), diffuse regional wall-motion abnormalities, severe mitral regurgitation, and moderate tricuspid regurgitation consistent with an exacerbation of heart failure.

We considered the possibility that her heart failure symptoms might be due to precipitous up-titration of her levothyroxine dose, given her borderline-elevated free thyroxine (T4) and increase in cardiac index (currently 4.45 L/min/m2, previously 2.20 L/min/m2 by the left ventricular outflow tract velocity time integral method). However, given her reduced ejection fraction, this clinical presentation most likely represented an acute exacerbation of her chronic heart failure. Her subjective fevers were thought to be due to a viral infection of the upper respiratory tract. The macrocytic anemia and thrombocytopenia were thought to be a side effect of her long-standing treatment with hydroxyurea for essential thrombocythemia, although amiodarone has also been associated with cytopenia.8

Treatment was started with intravenous diuretics and positive pressure ventilation with oxygen supplementation. Her levothyroxine dose was reduced, and her hydroxyurea was stopped.

Chest computed tomography axial views
Figure 1. Chest computed tomography axial views demonstrated increased attenuation in the liver (A, arrow) and left pleural base (B, arrow). Evaluation of the right lung base revealed ground-glass opacities (C, arrow) and honeycombing (D, arrow). These findings were consistent with amiodarone pulmonary toxicity.
After aggressive diuresis, our patient returned to euvolemia. However, she had persistent fine crackles and hypoxia. She had no further fever, and her vital signs were otherwise stable. Her cytopenia improved with cessation of hydroxyurea. Chest computed tomography (CT) showed bibasilar ground-glass infiltrates with areas of interstitial fibrosis, high-attenuation pleural lesions, and increased liver attenuation (Figure 1).

Further testing for connective tissue disease and hypersensitivity pneumonitis was also done, and the results were negative. To exclude an atypical infection, bronchoalveolar lavage was performed; preliminary microbial testing was negative, and the white blood cell count in the lavage fluid was 90% macrophages (pigment-laden), 7% neutrophils, and 3% lymphocytes.

WHAT IS THE CAUSE OF HER PERSISTENT PULMONARY FINDINGS?

2. Given the CT findings and laboratory results, what is the most likely cause of our patient’s persistent crackles and hypoxia?

  • Heart failure with reduced ejection fraction
  • Bacterial pneumonia
  • Idiopathic pulmonary fibrosis
  • Amiodarone pulmonary toxicity

Heart failure with reduced ejection fraction can cause ground-glass opacities on CT due to increased pulmonary edema. Although our patient initially presented with acute decompensation of heart failure with reduced ejection fraction decompensation, she had returned to euvolemia after aggressive diuresis. Moreover, increased pleural and liver attenuation are not typically seen as a result of heart failure with reduced ejection fraction, making this diagnosis less likely.

Bacterial pneumonia typically presents with cough, fever, and purulent sputum production.9 Further evaluation usually reveals decreased breath sounds, dullness to percussion, and leukocytosis.10 Chest CT in bacterial pneumonia commonly shows a focal area of consolidation, which was not seen in our patient.11

Idiopathic pulmonary fibrosis usually presents with slowly progressive dyspnea and nonproductive cough.12 Physical examination usually reveals fine crackles and occasionally end-inspiratory “squeaks” if traction bronchiectasis is present.12 The diagnosis of idiopathic pulmonary fibrosis requires chest CT findings compatible with it (ie, basal fibrosis, reticular abnormalities, and honeycombing). However, it remains a diagnosis of exclusion and requires ruling out conditions known to cause pulmonary fibrosis such as hypersensitivity pneumonitis, connective tissue disease, and certain medications.12

Although idiopathic pulmonary fibrosis remained in the differential diagnosis, our patient remained on amiodarone, a known cause of pulmonary fibrosis.13 Similarly, the high-attenuation pleural lesions likely represented organizing pneumonia, which is more common in amiodarone pulmonary toxicity. And the ground-glass opacities made idiopathic pulmonary fibrosis unlikely, although they may be seen in an acute exacerbation of this disease.14 Thus, a diagnosis of idiopathic pulmonary fibrosis could not be made definitively.

Amiodarone pulmonary toxicity most commonly presents with acute to subacute cough and progressive dyspnea.13 Physical findings are similar to those in idiopathic pulmonary fibrosis and commonly include bibasilar crackles. Chest CT shows diffuse ground-glass opacities, reticular abnormalities, fibrosis, and increased attenuation of multiple organs, including the lungs, liver, and spleen.14 Bronchoalveolar lavage findings of lipid-laden macrophages suggest but do not definitively diagnose amiodarone pulmonary toxicity.15 And patients with acute amiodarone pulmonary toxicity may present with pigment-laden macrophages on bronchoalveolar lavage, as in our patient.16

Exclusion of hypersensitivity pneumonitis, connective tissue disease, and infection made our patient’s progressive dyspnea and chest CT findings of ground-glass opacities, fibrosis, and increased pulmonary and liver attenuation most consistent with amiodarone pulmonary toxicity.

Amiodarone was therefore discontinued. However, the test result of her lavage fluid for influenza A by polymerase chain reaction came back positive a few hours later.

 

 

WHAT IS THE NEXT STEP?

3. Given the positive influenza A polymerase chain reaction test, which of the following is the best next step in this patient’s management?

  • Surgical lung biopsy
  • Stop amiodarone and start supportive influenza management
  • Stop amiodarone and start dronedarone
  • Start an intravenous corticosteroid

Surgical lung biopsy is typically not required for diagnosis in patients with suspected amiodarone pulmonary toxicity. In addition, acute respiratory distress syndrome has been documented in patients who have undergone surgical biopsy for suspected amiodarone pulmonary toxicity.17

Thus, surgical biopsy is typically only done in cases of persistent symptoms despite withdrawal of amiodarone and initiation of steroid therapy.

Stopping amiodarone and starting supportive influenza management are the best next steps, as our patient’s fevers, cough, dyspnea, and laboratory test results were consistent with influenza.18 Moreover, CT findings of ground-glass opacities and reticular abnormalities can be seen in influenza.19

However, concomitant amiodarone pulmonary toxicity could not be ruled out, as CT showed increased lung and liver attenuation and fibrosis that could not be explained by influenza. And the elevation in aminotransferase levels more than 2 times the upper limit of normal and CT findings of increased liver attenuation suggested amiodarone hepatotoxicity. However, definitive diagnosis would require exclusion of other causes such as congestive hepatopathy, in some cases with liver biopsy.13

Our patient’s persistent hypoxia was thought to be due in part to influenza, and thus the best next step in management was to stop amiodarone and provide supportive care for influenza.

Dronedarone is an antiarrhythmic drug structurally and functionally similar to amiodarone. There are far fewer reports of pulmonary toxicity with dronedarone than with amiodarone.20 However, lack of data on dronedarone in amiodarone pulmonary toxicity, increased rates of hospitalization and death associated with dronedarone in patients like ours with advanced heart failure, and our patient’s previously implanted cardioverter-defibrillator for recurrent ventricular tachycardia all made dronedarone an undesirable alternative to amiodarone.21

Corticosteroids are useful in the treatment of amiodarone pulmonary toxicity when hypoxia and dyspnea are present at diagnosis.13 Our patient’s hypoxia and dyspnea were thought to be due in part to her acute influenza infection, and therefore corticosteroids were not used at the outset.

However, concomitant amiodarone pulmonary toxicity could not be excluded, and the elevation in aminotransferases of more than 2 times the upper limit of normal and CT findings of increased liver attenuation suggested amiodarone hepatotoxicity—though congestive hepatopathy remained in the differential diagnosis. Therefore, supportive therapy for influenza was instituted, and amiodarone was withheld. Her condition subsequently improved, and she was discharged.

FOLLOW-UP 1 MONTH LATER

At a follow-up visit 1 month later, our patient continued to have dyspnea and hypoxia. She did not have signs or symptoms consistent with decompensated heart failure.

Pulmonary function testing revealed the following values:

  • FEV1 0.69 L (56% of predicted)
  • FVC 1.08 L (64% of predicted)
  • Chest computed tomography
    Figure 2. In A, repeat chest computed tomography demonstrated increased liver attenuation (arrow); in B, it showed persistent ground-glass opacities (white arrow), increased pulmonary attenuation (black arrowhead), and worsening pleural effusions (black arrows). These findings supported the diagnosis of amiodarone pulmonary toxicity.
    FEV1/FVC ratio 64%
  • DLCO 2.20 mL/min/mm Hg (12% of predicted).

Aminotransferase levels had also normalized. Repeat chest CT showed persistent bibasilar interstitial fibrotic changes, enlarging bilateral pleural effusions, and persistent peripheral ground-glass opacities (Figure 2).

 

 

WHAT FURTHER TREATMENT IS APPROPRIATE?

4. Given the chest CT findings, which of the following is the most appropriate treatment strategy for this patient?

  • No further management, continue to hold amiodarone
  • Corticosteroids
  • Repeat bronchoalveolar lavage
  • Intravenous antibiotics

No further management of amiodarone pulmonary toxicity would be appropriate if our patient did not have a high burden of symptoms. However, when patients with amiodarone pulmonary toxicity present with hypoxia and dyspnea, corticosteroids should be started.13 Our patient remained symptomatic after discontinuation of amiodarone and resolution of her influenza infection, and CT showed persistent signs of amiodarone pulmonary toxicity, which required further management.

Corticosteroids are useful in treating amiodarone pulmonary toxicity when hypoxia and dyspnea are present at diagnosis. Our patient’s persistent ground-glass opacities, fibrotic changes, and increased attenuation in multiple organs on CT, coupled with a confirmed reduction in FVC of greater than 15% and reduction in DLCO of greater than 20% after recovery from influenza, were most consistent with persistent amiodarone pulmonary toxicity.13

Although our patient’s amiodarone had been discontinued, the long half-life of the drug (45 days) allowed pulmonary toxicity to progress even after the drug was discontinued.22 Because our patient continued to have hypoxia and dyspnea on exertion, the most appropriate next step in management (in addition to managing her pleural effusions) was to start corticosteroids.

For amiodarone pulmonary toxicity, prednisone is typically started at 40 to 60 mg daily and can result in rapid improvement in symptoms.13 Tapering should be slow and may take several months.

Bronchoalveolar lavage is typically used in suspected cases of amiodarone pulmonary toxicity only to rule out an alternative diagnosis such as infection. Lipid-laden macrophages may be seen in the fluid. However, lipid-laden macrophages are not diagnostic of amiodarone pulmonary toxicity, as this finding may also be seen in patients taking amiodarone who do not develop pulmonary toxicity.15 Other findings on bronchoalveolar lavage in amiodarone pulmonary toxicity are nonspecific and are not diagnostically useful.13

Intravenous antibiotics are appropriate if bacterial pneumonia is suspected. However, bacterial pneumonia typically presents with cough, fever, purulent sputum production, and focal consolidation on chest imaging.9 Our patient’s CT findings of persistent peripheral ground-glass opacities and lack of cough, fever, or purulent sputum production were not consistent with bacterial pneumonia, and therefore intravenous antibiotics were not indicated.

CASE CONCLUSION

Given our patient’s persistent dyspnea, hypoxia, and chest CT findings consistent with amiodarone pulmonary toxicity, it was recommended that she start corticosteroids. However, before starting therapy, she suffered a femoral fracture that required surgical intervention. Around the time of the procedure, she had an ST-segment elevation myocardial infarction requiring vasopressor support and mechanical ventilation. At that time, the patient and family decided to pursue comfort measures, and she died peacefully.

MORE ABOUT AMIODARONE PULMONARY TOXICITY

Pulmonary toxicity is a well-described consequence of amiodarone therapy.23 Amiodarone carries a 2% risk of pulmonary toxicity.24 Although higher doses are more likely to cause pulmonary toxicity, lower doses also have been implicated.22,24 Preexisting pulmonary disease may predispose patients taking amiodarone to pulmonary toxicity; however, this is not uniformly seen.25

Mortality rates as high as 10% from amiodarone pulmonary toxicity have been reported. Thus, diligent surveillance for pulmonary toxicity with pulmonary function tests in patients taking amiodarone is mandatory. In particular, a reduction in FVC of greater than 15% or in DLCO of greater than 20% from baseline may be seen in amiodarone pulmonary toxicity.26

Amiodarone pulmonary toxicity can present at any time after the start of therapy, but it occurs most often after 6 to 12 months.13 Patients typically experience insidious dyspnea; however, presentation with acute to subacute cough and progressive dyspnea can occur, especially with high concentrations of supplemental oxygen with or without mechanical ventilation.12,27 Findings on physical examination include bibasilar crackles. CT chest findings include diffuse ground-glass opacities, reticular abnormalities, fibrosis, and increased attenuation in multiple organs, including the lung, liver, and spleen.14

The diagnosis of amiodarone pulmonary toxicity requires ruling out hypersensitivity pneumonitis, connective tissue disease, heart failure, and infection. Surgical biopsy and bronchoalveolar lavage are not commonly used to establish the diagnosis of amiodarone pulmonary toxicity, as surgical biopsy increases the risk of acute respiratory distress syndrome, and the results of bronchoalveolar lavage are usually nonspecific.13,15

Initial treatment involves discontinuing the amiodarone once the diagnosis is suspected. If patients have worsening hypoxia or dyspnea at the time of diagnosis, corticosteroids can be used. Prednisone is typically started at 40 to 60 mg daily and can result in rapid improvement in symptoms.13 Tapering of corticosteroids should occur slowly and may take several months.

An 87-year-old woman was brought to the intensive care unit with worsening shortness of breath on exertion, fatigue, orthopnea, paroxysmal nocturnal dyspnea, lower extremity swelling, subjective fever, productive cough, and rhinorrhea over the last week. She reported no chest pain, lightheadedness, or palpitations. Her medical history included the following:

  • Coronary artery disease requiring coronary artery bypass grafting
  • Ischemic cardiomyopathy
  • Severe mitral regurgitation
  • Moderate tricuspid regurgitation
  • Pulmonary hypertension
  • Cardiac arrest with recurrent ventricular tachycardia requiring an implanted cardioverter-defibrillator and amiodarone therapy
  • Hypothyroidism requiring levothyroxine
  • Asthma with a moderate obstructive pattern: forced expiratory volume in 1 second (FEV1) 60% of predicted, forced vital capacity (FVC) 2.06 L, FEV1/FVC 54%, diffusing capacity for carbon monoxide (DLCO) 72% of predicted with positive bronchodilator response
  • Long-standing essential thrombocythemia treated with hydroxyurea.

Before admission, she had been reliably taking guideline-directed heart failure therapy as well as amiodarone for her recurrent ventricular tachycardia. Her levothyroxine had recently been increased as well.

The patient's laboratory values
Physical examination. On admission, her blood pressure was 95/53 mm Hg, heart rate 73 beats per minute, temperature 36.7ºC (98.1ºF), and oxygen saturation 81% requiring supplemental oxygen 15 L/min by nonrebreather face mask. Physical examination revealed elevated jugular venous pressure, bibasilar crackles, lower extremity edema, and a grade 3 of 6 holosystolic murmur both at the left sternal border and at the apex radiating to the axilla. There was no evidence of wheezing or pulsus paradoxus.

Initial laboratory evaluation revealed abnormal values (Table 1).

Electrocardiography showed sinus rhythm and an old left bundle branch block.

Chest radiography showed cardiomegaly, bilateral pleural effusions, and pulmonary edema.

WHAT IS THE CAUSE OF HER SYMPTOMS?

1. Based on the available information, which of the following is the most likely cause of this patient’s clinical presentation?

  • Acute decompensated heart failure
  • Pulmonary embolism
  • Exacerbation of asthma
  • Exacerbation of chronic obstructive pulmonary disease (COPD)

Heart failure is a clinical diagnosis based on careful history-taking and physical examination. Major criteria include paroxysmal nocturnal dyspnea, orthopnea, elevated jugular venous pressure, pulmonary crackles, a third heart sound, cardiomegaly, pulmonary edema, and weight loss of more than 4.5 kg with diuretic therapy.1 N-terminal pro-B-type natriuretic peptide (NT-proBNP) is also an effective marker of acute decompensated heart failure in the proper clinical setting.2

Our patient’s elevated jugular venous pressure, bibasilar crackles, lower extremity edema, chest radiography findings consistent with pulmonary edema, markedly elevated NT-proBNP, history of orthopnea, paroxysmal nocturnal dyspnea, and dyspnea on exertion were most consistent with acute decompensated heart failure. Her cough and subjective fevers were thought to be due to an upper respiratory tract viral infection.

Pulmonary embolism causes pleuritic chest pain, dyspnea, and, occasionally, elevated troponin. The most common feature on electrocardiography is sinus tachycardia; nonspecific ST-segment and T-wave changes may also be seen.3

Although pulmonary embolism remained in the differential diagnosis, our patient’s lack of typical features of pulmonary embolism made this less likely.

Asthma is characterized by recurrent airflow obstruction and bronchial hyperresponsiveness.4 Asthma exacerbations present with wheezing, tachypnea, tachycardia, and pulsus paradoxus.5

Despite her previous asthma diagnosis, our patient’s lack of typical features of asthma exacerbation made this diagnosis unlikely.

COPD exacerbations present with increased dyspnea, cough, sputum production, wheezing, lung resonance to percussion, and distant heart sounds, and are characterized by airflow obstruction.6,7

Although our patient presented with cough and dyspnea, she had no history of COPD and her other signs and symptoms (elevated jugular venous pressure, elevated NT-proBNP, and peripheral edema) could not be explained by COPD exacerbation.

 

 

OUR PATIENT UNDERWENT FURTHER TESTING

Echocardiography revealed severe left ventricular enlargement, an ejection fraction of 20% (which was near her baseline value), diffuse regional wall-motion abnormalities, severe mitral regurgitation, and moderate tricuspid regurgitation consistent with an exacerbation of heart failure.

We considered the possibility that her heart failure symptoms might be due to precipitous up-titration of her levothyroxine dose, given her borderline-elevated free thyroxine (T4) and increase in cardiac index (currently 4.45 L/min/m2, previously 2.20 L/min/m2 by the left ventricular outflow tract velocity time integral method). However, given her reduced ejection fraction, this clinical presentation most likely represented an acute exacerbation of her chronic heart failure. Her subjective fevers were thought to be due to a viral infection of the upper respiratory tract. The macrocytic anemia and thrombocytopenia were thought to be a side effect of her long-standing treatment with hydroxyurea for essential thrombocythemia, although amiodarone has also been associated with cytopenia.8

Treatment was started with intravenous diuretics and positive pressure ventilation with oxygen supplementation. Her levothyroxine dose was reduced, and her hydroxyurea was stopped.

Chest computed tomography axial views
Figure 1. Chest computed tomography axial views demonstrated increased attenuation in the liver (A, arrow) and left pleural base (B, arrow). Evaluation of the right lung base revealed ground-glass opacities (C, arrow) and honeycombing (D, arrow). These findings were consistent with amiodarone pulmonary toxicity.
After aggressive diuresis, our patient returned to euvolemia. However, she had persistent fine crackles and hypoxia. She had no further fever, and her vital signs were otherwise stable. Her cytopenia improved with cessation of hydroxyurea. Chest computed tomography (CT) showed bibasilar ground-glass infiltrates with areas of interstitial fibrosis, high-attenuation pleural lesions, and increased liver attenuation (Figure 1).

Further testing for connective tissue disease and hypersensitivity pneumonitis was also done, and the results were negative. To exclude an atypical infection, bronchoalveolar lavage was performed; preliminary microbial testing was negative, and the white blood cell count in the lavage fluid was 90% macrophages (pigment-laden), 7% neutrophils, and 3% lymphocytes.

WHAT IS THE CAUSE OF HER PERSISTENT PULMONARY FINDINGS?

2. Given the CT findings and laboratory results, what is the most likely cause of our patient’s persistent crackles and hypoxia?

  • Heart failure with reduced ejection fraction
  • Bacterial pneumonia
  • Idiopathic pulmonary fibrosis
  • Amiodarone pulmonary toxicity

Heart failure with reduced ejection fraction can cause ground-glass opacities on CT due to increased pulmonary edema. Although our patient initially presented with acute decompensation of heart failure with reduced ejection fraction decompensation, she had returned to euvolemia after aggressive diuresis. Moreover, increased pleural and liver attenuation are not typically seen as a result of heart failure with reduced ejection fraction, making this diagnosis less likely.

Bacterial pneumonia typically presents with cough, fever, and purulent sputum production.9 Further evaluation usually reveals decreased breath sounds, dullness to percussion, and leukocytosis.10 Chest CT in bacterial pneumonia commonly shows a focal area of consolidation, which was not seen in our patient.11

Idiopathic pulmonary fibrosis usually presents with slowly progressive dyspnea and nonproductive cough.12 Physical examination usually reveals fine crackles and occasionally end-inspiratory “squeaks” if traction bronchiectasis is present.12 The diagnosis of idiopathic pulmonary fibrosis requires chest CT findings compatible with it (ie, basal fibrosis, reticular abnormalities, and honeycombing). However, it remains a diagnosis of exclusion and requires ruling out conditions known to cause pulmonary fibrosis such as hypersensitivity pneumonitis, connective tissue disease, and certain medications.12

Although idiopathic pulmonary fibrosis remained in the differential diagnosis, our patient remained on amiodarone, a known cause of pulmonary fibrosis.13 Similarly, the high-attenuation pleural lesions likely represented organizing pneumonia, which is more common in amiodarone pulmonary toxicity. And the ground-glass opacities made idiopathic pulmonary fibrosis unlikely, although they may be seen in an acute exacerbation of this disease.14 Thus, a diagnosis of idiopathic pulmonary fibrosis could not be made definitively.

Amiodarone pulmonary toxicity most commonly presents with acute to subacute cough and progressive dyspnea.13 Physical findings are similar to those in idiopathic pulmonary fibrosis and commonly include bibasilar crackles. Chest CT shows diffuse ground-glass opacities, reticular abnormalities, fibrosis, and increased attenuation of multiple organs, including the lungs, liver, and spleen.14 Bronchoalveolar lavage findings of lipid-laden macrophages suggest but do not definitively diagnose amiodarone pulmonary toxicity.15 And patients with acute amiodarone pulmonary toxicity may present with pigment-laden macrophages on bronchoalveolar lavage, as in our patient.16

Exclusion of hypersensitivity pneumonitis, connective tissue disease, and infection made our patient’s progressive dyspnea and chest CT findings of ground-glass opacities, fibrosis, and increased pulmonary and liver attenuation most consistent with amiodarone pulmonary toxicity.

Amiodarone was therefore discontinued. However, the test result of her lavage fluid for influenza A by polymerase chain reaction came back positive a few hours later.

 

 

WHAT IS THE NEXT STEP?

3. Given the positive influenza A polymerase chain reaction test, which of the following is the best next step in this patient’s management?

  • Surgical lung biopsy
  • Stop amiodarone and start supportive influenza management
  • Stop amiodarone and start dronedarone
  • Start an intravenous corticosteroid

Surgical lung biopsy is typically not required for diagnosis in patients with suspected amiodarone pulmonary toxicity. In addition, acute respiratory distress syndrome has been documented in patients who have undergone surgical biopsy for suspected amiodarone pulmonary toxicity.17

Thus, surgical biopsy is typically only done in cases of persistent symptoms despite withdrawal of amiodarone and initiation of steroid therapy.

Stopping amiodarone and starting supportive influenza management are the best next steps, as our patient’s fevers, cough, dyspnea, and laboratory test results were consistent with influenza.18 Moreover, CT findings of ground-glass opacities and reticular abnormalities can be seen in influenza.19

However, concomitant amiodarone pulmonary toxicity could not be ruled out, as CT showed increased lung and liver attenuation and fibrosis that could not be explained by influenza. And the elevation in aminotransferase levels more than 2 times the upper limit of normal and CT findings of increased liver attenuation suggested amiodarone hepatotoxicity. However, definitive diagnosis would require exclusion of other causes such as congestive hepatopathy, in some cases with liver biopsy.13

Our patient’s persistent hypoxia was thought to be due in part to influenza, and thus the best next step in management was to stop amiodarone and provide supportive care for influenza.

Dronedarone is an antiarrhythmic drug structurally and functionally similar to amiodarone. There are far fewer reports of pulmonary toxicity with dronedarone than with amiodarone.20 However, lack of data on dronedarone in amiodarone pulmonary toxicity, increased rates of hospitalization and death associated with dronedarone in patients like ours with advanced heart failure, and our patient’s previously implanted cardioverter-defibrillator for recurrent ventricular tachycardia all made dronedarone an undesirable alternative to amiodarone.21

Corticosteroids are useful in the treatment of amiodarone pulmonary toxicity when hypoxia and dyspnea are present at diagnosis.13 Our patient’s hypoxia and dyspnea were thought to be due in part to her acute influenza infection, and therefore corticosteroids were not used at the outset.

However, concomitant amiodarone pulmonary toxicity could not be excluded, and the elevation in aminotransferases of more than 2 times the upper limit of normal and CT findings of increased liver attenuation suggested amiodarone hepatotoxicity—though congestive hepatopathy remained in the differential diagnosis. Therefore, supportive therapy for influenza was instituted, and amiodarone was withheld. Her condition subsequently improved, and she was discharged.

FOLLOW-UP 1 MONTH LATER

At a follow-up visit 1 month later, our patient continued to have dyspnea and hypoxia. She did not have signs or symptoms consistent with decompensated heart failure.

Pulmonary function testing revealed the following values:

  • FEV1 0.69 L (56% of predicted)
  • FVC 1.08 L (64% of predicted)
  • Chest computed tomography
    Figure 2. In A, repeat chest computed tomography demonstrated increased liver attenuation (arrow); in B, it showed persistent ground-glass opacities (white arrow), increased pulmonary attenuation (black arrowhead), and worsening pleural effusions (black arrows). These findings supported the diagnosis of amiodarone pulmonary toxicity.
    FEV1/FVC ratio 64%
  • DLCO 2.20 mL/min/mm Hg (12% of predicted).

Aminotransferase levels had also normalized. Repeat chest CT showed persistent bibasilar interstitial fibrotic changes, enlarging bilateral pleural effusions, and persistent peripheral ground-glass opacities (Figure 2).

 

 

WHAT FURTHER TREATMENT IS APPROPRIATE?

4. Given the chest CT findings, which of the following is the most appropriate treatment strategy for this patient?

  • No further management, continue to hold amiodarone
  • Corticosteroids
  • Repeat bronchoalveolar lavage
  • Intravenous antibiotics

No further management of amiodarone pulmonary toxicity would be appropriate if our patient did not have a high burden of symptoms. However, when patients with amiodarone pulmonary toxicity present with hypoxia and dyspnea, corticosteroids should be started.13 Our patient remained symptomatic after discontinuation of amiodarone and resolution of her influenza infection, and CT showed persistent signs of amiodarone pulmonary toxicity, which required further management.

Corticosteroids are useful in treating amiodarone pulmonary toxicity when hypoxia and dyspnea are present at diagnosis. Our patient’s persistent ground-glass opacities, fibrotic changes, and increased attenuation in multiple organs on CT, coupled with a confirmed reduction in FVC of greater than 15% and reduction in DLCO of greater than 20% after recovery from influenza, were most consistent with persistent amiodarone pulmonary toxicity.13

Although our patient’s amiodarone had been discontinued, the long half-life of the drug (45 days) allowed pulmonary toxicity to progress even after the drug was discontinued.22 Because our patient continued to have hypoxia and dyspnea on exertion, the most appropriate next step in management (in addition to managing her pleural effusions) was to start corticosteroids.

For amiodarone pulmonary toxicity, prednisone is typically started at 40 to 60 mg daily and can result in rapid improvement in symptoms.13 Tapering should be slow and may take several months.

Bronchoalveolar lavage is typically used in suspected cases of amiodarone pulmonary toxicity only to rule out an alternative diagnosis such as infection. Lipid-laden macrophages may be seen in the fluid. However, lipid-laden macrophages are not diagnostic of amiodarone pulmonary toxicity, as this finding may also be seen in patients taking amiodarone who do not develop pulmonary toxicity.15 Other findings on bronchoalveolar lavage in amiodarone pulmonary toxicity are nonspecific and are not diagnostically useful.13

Intravenous antibiotics are appropriate if bacterial pneumonia is suspected. However, bacterial pneumonia typically presents with cough, fever, purulent sputum production, and focal consolidation on chest imaging.9 Our patient’s CT findings of persistent peripheral ground-glass opacities and lack of cough, fever, or purulent sputum production were not consistent with bacterial pneumonia, and therefore intravenous antibiotics were not indicated.

CASE CONCLUSION

Given our patient’s persistent dyspnea, hypoxia, and chest CT findings consistent with amiodarone pulmonary toxicity, it was recommended that she start corticosteroids. However, before starting therapy, she suffered a femoral fracture that required surgical intervention. Around the time of the procedure, she had an ST-segment elevation myocardial infarction requiring vasopressor support and mechanical ventilation. At that time, the patient and family decided to pursue comfort measures, and she died peacefully.

MORE ABOUT AMIODARONE PULMONARY TOXICITY

Pulmonary toxicity is a well-described consequence of amiodarone therapy.23 Amiodarone carries a 2% risk of pulmonary toxicity.24 Although higher doses are more likely to cause pulmonary toxicity, lower doses also have been implicated.22,24 Preexisting pulmonary disease may predispose patients taking amiodarone to pulmonary toxicity; however, this is not uniformly seen.25

Mortality rates as high as 10% from amiodarone pulmonary toxicity have been reported. Thus, diligent surveillance for pulmonary toxicity with pulmonary function tests in patients taking amiodarone is mandatory. In particular, a reduction in FVC of greater than 15% or in DLCO of greater than 20% from baseline may be seen in amiodarone pulmonary toxicity.26

Amiodarone pulmonary toxicity can present at any time after the start of therapy, but it occurs most often after 6 to 12 months.13 Patients typically experience insidious dyspnea; however, presentation with acute to subacute cough and progressive dyspnea can occur, especially with high concentrations of supplemental oxygen with or without mechanical ventilation.12,27 Findings on physical examination include bibasilar crackles. CT chest findings include diffuse ground-glass opacities, reticular abnormalities, fibrosis, and increased attenuation in multiple organs, including the lung, liver, and spleen.14

The diagnosis of amiodarone pulmonary toxicity requires ruling out hypersensitivity pneumonitis, connective tissue disease, heart failure, and infection. Surgical biopsy and bronchoalveolar lavage are not commonly used to establish the diagnosis of amiodarone pulmonary toxicity, as surgical biopsy increases the risk of acute respiratory distress syndrome, and the results of bronchoalveolar lavage are usually nonspecific.13,15

Initial treatment involves discontinuing the amiodarone once the diagnosis is suspected. If patients have worsening hypoxia or dyspnea at the time of diagnosis, corticosteroids can be used. Prednisone is typically started at 40 to 60 mg daily and can result in rapid improvement in symptoms.13 Tapering of corticosteroids should occur slowly and may take several months.

References
  1. McKee PA, Castelli WP, McNamara PM, Kannel WB. The natural history of congestive heart failure: the Framingham study. N Engl J Med 1971; 285(26):1441–1446. doi:10.1056/NEJM197112232852601
  2. Stein PD, Terrin ML, Hales CA, et al. Clinical, laboratory, roentgenographic, and electrocardiographic findings in patients with acute pulmonary embolism and no pre-existing cardiac or pulmonary disease. Chest 1991; 100(3):598–603. pmid:1909617
  3. Baggish AL, Siebert U, Lainchbury JG, et al. A validated clinical and biochemical score for the diagnosis of acute heart failure: the ProBNP investigation of dyspnea in the emergency department (PRIDE) acute heart failure score. Am Heart J 2006; 151(1):48–54. doi:10.1016/j.ahj.2005.02.031
  4. National Heart, Lung, and Blood Institute. National Asthma Education and Prevention Program. Expert panel report 3: Guidelines for the diagnosis and management of asthma. www.nhlbi.nih.gov/sites/default/files/media/docs/asthgdln_1.pdf. Accessed August 3, 2018.
  5. Brenner BE, Abraham E, Simon RR. Position and diaphoresis in acute asthma. Am J Med 1983; 74(6):1005–1009. pmid:6407304
  6. Badgett RG, Tanaka DJ, Hunt DK, et al. Can moderate chronic obstructive pulmonary disease be diagnosed by historical and physical findings alone? Am J Med 1993; 94(2):188–196. pmid:8430714
  7. Global Initiative for Chronic Obstructive Lung Disease (GOLD). Global strategy for the diagnosis, management, and prevention of chronic obstructive pulmonary disease. https://goldcopd.org/wp-content/uploads/2017/11/GOLD-2018-v6.0-FINAL-revised-20-Nov_WMS.pdf. Accessed August 17, 2018.
  8. Erie AJ, McClure RF, Wolanskyj AP. Amiodarone-induced bone marrow granulomas: an unusual cause of reversible pancytopenia. Hematol Rep 2010; 2(1):e6. doi:10.4081/hr.2010.e6
  9. Marrie TJ. Community-acquired pneumonia. Clin Infect Dis 1994; 18(4):501–513. pmid:8038304
  10. Metlay JP, Kapoor WN, Fine MJ. Does this patient have community-acquired pneumonia? Diagnosing pneumonia by history and physical examination. JAMA 1997; 278(17):1440–1445. pmid:9356004
  11. Walker CM, Abbott GF, Greene RE, Shepard JA, Vummidi D, Digumarthy SR. Imaging pulmonary infection: classic signs and patterns. AJR Am J Roentgenol 2014; 202(3):479–492. doi:10.2214/AJR.13.11463
  12. Raghu G, Collard HR, Egan JJ, et al; ATS/ERS/JRS/ALAT Committee on Idiopathic Pulmonary Fibrosis. An official ATS/ERS/JRS/ALAT statement: idiopathic pulmonary fibrosis: evidence-based guidelines for diagnosis and management. Am J Respir Crit Care Med 2011; 183(6):788–824. doi:10.1164/rccm.2009-040GL
  13. Goldschlager N, Epstein AE, Naccarelli GV, et al; Practice Guidelines Sub-committee, North American Society of Pacing and Electrophysiology (HRS). A practical guide for clinicians who treat patients with amiodarone: 2007. Heart Rhythm 2007; 4(9):1250–1259. doi:10.1016/j.hrthm.2007.07.020
  14. Kuhlman JE, Teigen C, Ren H, Hruban RH, Hutchins GM, Fishman EK. Amiodarone pulmonary toxicity: CT findings in symptomatic patients. Radiology 1990; 177(1):121–125. doi:10.1148/radiology.177.1.2399310
  15. Martin WJ 2nd, Rosenow EC 3rd. Amiodarone pulmonary toxicity: recognition and pathogenesis (Part I). Chest 1988; 93(5):1067–1075. pmid:3282816
  16. Iskandar SB, Abi-Saleh B, Keith RL, Byrd RP Jr, Roy TM. Amiodarone-induced alveolar hemorrhage. South Med J 2006; 99(4):383–387.
  17. Van Mieghem W, Coolen L, Malysse I, Lacquet LM, Deneffe GJ, Demedts MG. Amiodarone and the development of ARDS after lung surgery. Chest 1994; 105(6):1642–1645. pmid:8205854
  18. Nicholson KG. Clinical features of influenza. Semin Respir Infect 1992; 7(1):26–37. pmid:1609165
  19. Muller NL, Franquet T, Lee KS, Silva CIS. Viruses, mycoplasma, and chlamydia. In: Imaging of Pulmonary Infections. Philadelphia, PA: Lippincott Williams & Wilkins; 2007:94–114.
  20. Stack S, Nguyen DV, Casto A, Ahuja N. Diffuse alveolar damage in a patient receiving dronedarone. Chest 2015; 147(4):e131–e133. doi:10.1378/chest.14-1849
  21. De Ferrari GM, Dusi V. Drug safety evaluation of dronedarone in atrial fibrillation. Expert Opin Drug Saf 2012; 11(6):1023–1045. doi:10.1517/14740338.2012.722994
  22. Okayasu K, Takeda Y, Kojima J, et al. Amiodarone pulmonary toxicity: a patient with three recurrences of pulmonary toxicity and consideration of the probable risk for relapse. Intern Med 2006; 45(22):1303–1307. pmid:17170505
  23. Vorperian VR, Havighurst TC, Miller S, January CT. Adverse effects of low dose amiodarone: a meta-analysis. J Am Coll Cardiol 1997; 30(3):791–798. pmid:9283542
  24. Amiodarone Trials Meta-Analysis Investigators. Effect of prophylactic amiodarone on mortality after acute myocardial infarction and in congestive heart failure: meta-analysis of individual data from 6500 patients in randomised trials. Lancet 1997; 350(9089):1417–1424. pmid:9371164
  25. Olshansky B, Sami M, Rubin A, et al; NHLBI AFFIRM Investigators. Use of amiodarone for atrial fibrillation in patients with preexisting pulmonary disease in the AFFIRM study. Am J Cardiol 2005; 95(3):404–405. doi:10.1016/j.amjcard.2004.09.044
  26. Camus P. Interstitial lung disease from drugs, biologics, and radiation. In: Schwartz MI, King TE Jr, eds. Interstitial Lung Disease. 5th ed. Shelton, CT: People’s Medical Publishing House; 2011:637–644.
  27. Wolkove N, Baltzan M. Amiodarone pulmonary toxicity. Can Respir J 2009; 16(2):43–48. doi:10.1155/2009/282540
References
  1. McKee PA, Castelli WP, McNamara PM, Kannel WB. The natural history of congestive heart failure: the Framingham study. N Engl J Med 1971; 285(26):1441–1446. doi:10.1056/NEJM197112232852601
  2. Stein PD, Terrin ML, Hales CA, et al. Clinical, laboratory, roentgenographic, and electrocardiographic findings in patients with acute pulmonary embolism and no pre-existing cardiac or pulmonary disease. Chest 1991; 100(3):598–603. pmid:1909617
  3. Baggish AL, Siebert U, Lainchbury JG, et al. A validated clinical and biochemical score for the diagnosis of acute heart failure: the ProBNP investigation of dyspnea in the emergency department (PRIDE) acute heart failure score. Am Heart J 2006; 151(1):48–54. doi:10.1016/j.ahj.2005.02.031
  4. National Heart, Lung, and Blood Institute. National Asthma Education and Prevention Program. Expert panel report 3: Guidelines for the diagnosis and management of asthma. www.nhlbi.nih.gov/sites/default/files/media/docs/asthgdln_1.pdf. Accessed August 3, 2018.
  5. Brenner BE, Abraham E, Simon RR. Position and diaphoresis in acute asthma. Am J Med 1983; 74(6):1005–1009. pmid:6407304
  6. Badgett RG, Tanaka DJ, Hunt DK, et al. Can moderate chronic obstructive pulmonary disease be diagnosed by historical and physical findings alone? Am J Med 1993; 94(2):188–196. pmid:8430714
  7. Global Initiative for Chronic Obstructive Lung Disease (GOLD). Global strategy for the diagnosis, management, and prevention of chronic obstructive pulmonary disease. https://goldcopd.org/wp-content/uploads/2017/11/GOLD-2018-v6.0-FINAL-revised-20-Nov_WMS.pdf. Accessed August 17, 2018.
  8. Erie AJ, McClure RF, Wolanskyj AP. Amiodarone-induced bone marrow granulomas: an unusual cause of reversible pancytopenia. Hematol Rep 2010; 2(1):e6. doi:10.4081/hr.2010.e6
  9. Marrie TJ. Community-acquired pneumonia. Clin Infect Dis 1994; 18(4):501–513. pmid:8038304
  10. Metlay JP, Kapoor WN, Fine MJ. Does this patient have community-acquired pneumonia? Diagnosing pneumonia by history and physical examination. JAMA 1997; 278(17):1440–1445. pmid:9356004
  11. Walker CM, Abbott GF, Greene RE, Shepard JA, Vummidi D, Digumarthy SR. Imaging pulmonary infection: classic signs and patterns. AJR Am J Roentgenol 2014; 202(3):479–492. doi:10.2214/AJR.13.11463
  12. Raghu G, Collard HR, Egan JJ, et al; ATS/ERS/JRS/ALAT Committee on Idiopathic Pulmonary Fibrosis. An official ATS/ERS/JRS/ALAT statement: idiopathic pulmonary fibrosis: evidence-based guidelines for diagnosis and management. Am J Respir Crit Care Med 2011; 183(6):788–824. doi:10.1164/rccm.2009-040GL
  13. Goldschlager N, Epstein AE, Naccarelli GV, et al; Practice Guidelines Sub-committee, North American Society of Pacing and Electrophysiology (HRS). A practical guide for clinicians who treat patients with amiodarone: 2007. Heart Rhythm 2007; 4(9):1250–1259. doi:10.1016/j.hrthm.2007.07.020
  14. Kuhlman JE, Teigen C, Ren H, Hruban RH, Hutchins GM, Fishman EK. Amiodarone pulmonary toxicity: CT findings in symptomatic patients. Radiology 1990; 177(1):121–125. doi:10.1148/radiology.177.1.2399310
  15. Martin WJ 2nd, Rosenow EC 3rd. Amiodarone pulmonary toxicity: recognition and pathogenesis (Part I). Chest 1988; 93(5):1067–1075. pmid:3282816
  16. Iskandar SB, Abi-Saleh B, Keith RL, Byrd RP Jr, Roy TM. Amiodarone-induced alveolar hemorrhage. South Med J 2006; 99(4):383–387.
  17. Van Mieghem W, Coolen L, Malysse I, Lacquet LM, Deneffe GJ, Demedts MG. Amiodarone and the development of ARDS after lung surgery. Chest 1994; 105(6):1642–1645. pmid:8205854
  18. Nicholson KG. Clinical features of influenza. Semin Respir Infect 1992; 7(1):26–37. pmid:1609165
  19. Muller NL, Franquet T, Lee KS, Silva CIS. Viruses, mycoplasma, and chlamydia. In: Imaging of Pulmonary Infections. Philadelphia, PA: Lippincott Williams & Wilkins; 2007:94–114.
  20. Stack S, Nguyen DV, Casto A, Ahuja N. Diffuse alveolar damage in a patient receiving dronedarone. Chest 2015; 147(4):e131–e133. doi:10.1378/chest.14-1849
  21. De Ferrari GM, Dusi V. Drug safety evaluation of dronedarone in atrial fibrillation. Expert Opin Drug Saf 2012; 11(6):1023–1045. doi:10.1517/14740338.2012.722994
  22. Okayasu K, Takeda Y, Kojima J, et al. Amiodarone pulmonary toxicity: a patient with three recurrences of pulmonary toxicity and consideration of the probable risk for relapse. Intern Med 2006; 45(22):1303–1307. pmid:17170505
  23. Vorperian VR, Havighurst TC, Miller S, January CT. Adverse effects of low dose amiodarone: a meta-analysis. J Am Coll Cardiol 1997; 30(3):791–798. pmid:9283542
  24. Amiodarone Trials Meta-Analysis Investigators. Effect of prophylactic amiodarone on mortality after acute myocardial infarction and in congestive heart failure: meta-analysis of individual data from 6500 patients in randomised trials. Lancet 1997; 350(9089):1417–1424. pmid:9371164
  25. Olshansky B, Sami M, Rubin A, et al; NHLBI AFFIRM Investigators. Use of amiodarone for atrial fibrillation in patients with preexisting pulmonary disease in the AFFIRM study. Am J Cardiol 2005; 95(3):404–405. doi:10.1016/j.amjcard.2004.09.044
  26. Camus P. Interstitial lung disease from drugs, biologics, and radiation. In: Schwartz MI, King TE Jr, eds. Interstitial Lung Disease. 5th ed. Shelton, CT: People’s Medical Publishing House; 2011:637–644.
  27. Wolkove N, Baltzan M. Amiodarone pulmonary toxicity. Can Respir J 2009; 16(2):43–48. doi:10.1155/2009/282540
Issue
Cleveland Clinic Journal of Medicine - 85(9)
Issue
Cleveland Clinic Journal of Medicine - 85(9)
Page Number
685-692
Page Number
685-692
Publications
Publications
Topics
Article Type
Display Headline
Shortness of breath, fever, cough, and more in an elderly woman
Display Headline
Shortness of breath, fever, cough, and more in an elderly woman
Legacy Keywords
amiodarone, pulmonary toxicity, lung toxicity, shortness of breath, heart failure, ventricular tachycardia, Brody Slostad, Megha Prasad, Nandan Anavekar
Legacy Keywords
amiodarone, pulmonary toxicity, lung toxicity, shortness of breath, heart failure, ventricular tachycardia, Brody Slostad, Megha Prasad, Nandan Anavekar
Sections
Disallow All Ads
Content Gating
No Gating (article Unlocked/Free)
Alternative CME
Disqus Comments
Default
Gate On Date
Tue, 08/28/2018 - 07:30
Un-Gate On Date
Tue, 08/28/2018 - 07:30
Use ProPublica
CFC Schedule Remove Status
Tue, 08/28/2018 - 07:30
Hide sidebar & use full width
render the right sidebar.
Article PDF Media